Re: [obm-l] OBM 88 Problema 6.

2023-12-31 Por tôpico Anderson Torres
Em dom., 31 de dez. de 2023 às 00:56, Pedro José  escreveu:
>
> Boa noite!
> Cláudio, minha preocupação é com a solução em si da equação.
> O problema original pede que demonstre que k é um quadrado perfeito. Todas 
> soluções que vi são baseadas nas relações de Girad ou Vieta's fórmula como 
> chamam lá fora.
> Eu parti do conhecimento de que k tem de ser quadrado perfeito.
> Consegui provar que tirando as soluções triviais a=0 ou b=0 ou a=b=1 
> b>=raiz(k)
> Aí achei a primeira solução para a equação, sem perda de generalidade, 
> considerei a>b, a=b só ocorre para a=b=1 ou a=b=0. Lá fora acho que nem 
> consideram 0 natural. Seguem a risca como foi o postulado de Peano.

O enunciado original dizia INTEIRO POSITIVO, e não "natural".
Os proponentes da IMO têm uma certa noção dessas pequenas polêmicas,
então eles costumam ser bastante verbosos sobre se 0 é considerado ou
não parte das soluções.
Curiosidade: na França 0 é considerado positivo E negativo ao mesmo tempo.
https://mathfour.com/arithmetic/is-zero-positive-or-negative

> Então para cada k=w^2 com w>1
> Tem um conjunto com uma sequência infinita de soluções.
> Sk={si=(ai,bi,k): i natural e i>=1| s1=(w^3,w,w^2) e si+1=(ai*w^2-bi, ai, 
> w^2).
> Consigo provar que todos termos da sequência são soluções.
> Não consigo provar que se há uma solução (a*,b*, k*) então (a*,b*,k*) ou 
> (b*,a*, k*) pertence a sequência Sk para k=w^2.

Ué, você pode imitar a solução do problema original. Se (a*,b*,K) é
solução E não está na rota dourada, então é possível encontrar uma
solução menor fora da rota dourada também, e assim por diante até
chegar na solução minimal. Mas a solução minimal é justo a que inicia
a rota dourada, absurdo.

> Eu não acho a solução da equação, só do problema como foi pedido, mostrar que 
> k é um QP, sem no entanto achar todas as soluções
>
> Cordialmente,
> PJMS
>
> Em sex., 29 de dez. de 2023 09:18, Claudio Buffara 
>  escreveu:
>>
>> Dá um Google em "IMO 88".
>> Vai ter até vídeo com a solução deste problema.
>>
>> On Thu, Dec 28, 2023 at 4:35 PM Pedro José  wrote:
>>>
>>> Boa tarde!
>>> Com referência a esse problema criei uma conjectura, não consegui provar 
>>> com a pretensão de abranger todas as soluções da equação:
>>>
>>> (a^2+b^2)/(ab+1)= k, com a,b,k Naturais e a>1, b>1 e k>1 Fiz essa restrição 
>>> para retirar as soluções triviais.
>>> E SPG considerei a>b, já que a=b só ocorre para a=b=1, que está fora pela 
>>> restrição acima e por ser uma equação simétrica em relação à a e b.
>>> O problema era provar que k era um quadrado perfeito.
>>> Gostaria de saber se alguém teria conhecimento da resolução em si do 
>>> problema, i.e., quais ternos (a*,b*,k*) são solução da equação.
>>> Caso ninguém tenha resolvido a equação, ainda, gostaria como faço para dar 
>>> divulgação da minha conjectura, onde tenho a pretenção de ter encontrado 
>>> todas as soluções possíveis para a equação em epígrafe, no Universo dos 
>>> Naturais, com a restrição a>1, b>1 e K>1.
>>>
>>> Agradeço quem puder me orientar.
>>>
>>> Cordialmente,
>>> PJMS
>>>
>>> --
>>> Esta mensagem foi verificada pelo sistema de antivírus e
>>> acredita-se estar livre de perigo.
>>
>>
>> --
>> Esta mensagem foi verificada pelo sistema de antivírus e
>> acredita-se estar livre de perigo.
>
>
> --
> Esta mensagem foi verificada pelo sistema de antivírus e
> acredita-se estar livre de perigo.

-- 
Esta mensagem foi verificada pelo sistema de antiv�rus e
 acredita-se estar livre de perigo.


=
Instru��es para entrar na lista, sair da lista e usar a lista em
http://www.mat.puc-rio.br/~obmlistas/obm-l.html
=


Re: [obm-l] OBM 88 Problema 6.

2023-12-30 Por tôpico Pedro José
Boa noite!
Cláudio, minha preocupação é com a solução em si da equação.
O problema original pede que demonstre que k é um quadrado perfeito. Todas
soluções que vi são baseadas nas relações de Girad ou Vieta's fórmula como
chamam lá fora.
Eu parti do conhecimento de que k tem de ser quadrado perfeito.
Consegui provar que tirando as soluções triviais a=0 ou b=0 ou a=b=1
b>=raiz(k)
Aí achei a primeira solução para a equação, sem perda de generalidade,
considerei a>b, a=b só ocorre para a=b=1 ou a=b=0. Lá fora acho que nem
consideram 0 natural. Seguem a risca como foi o postulado de Peano.
Então para cada k=w^2 com w>1
Tem um conjunto com uma sequência infinita de soluções.
Sk={si=(ai,bi,k): i natural e i>=1| s1=(w^3,w,w^2) e si+1=(ai*w^2-bi, ai,
w^2).
Consigo provar que todos termos da sequência são soluções.
Não consigo provar que se há uma solução (a*,b*, k*) então (a*,b*,k*) ou
(b*,a*, k*) pertence a sequência Sk para k=w^2.
Eu não acho a solução da equação, só do problema como foi pedido, mostrar
que k é um QP, sem no entanto achar todas as soluções

Cordialmente,
PJMS

Em sex., 29 de dez. de 2023 09:18, Claudio Buffara <
claudio.buff...@gmail.com> escreveu:

> Dá um Google em "IMO 88".
> Vai ter até vídeo com a solução deste problema.
>
> On Thu, Dec 28, 2023 at 4:35 PM Pedro José  wrote:
>
>> Boa tarde!
>> Com referência a esse problema criei uma conjectura, não consegui provar
>> com a pretensão de abranger todas as soluções da equação:
>>
>> (a^2+b^2)/(ab+1)= k, com a,b,k Naturais e a>1, b>1 e k>1 Fiz essa
>> restrição para retirar as soluções triviais.
>> E SPG considerei a>b, já que a=b só ocorre para a=b=1, que está fora pela
>> restrição acima e por ser uma equação simétrica em relação à a e b.
>> O problema era provar que k era um quadrado perfeito.
>> Gostaria de saber se alguém teria conhecimento da resolução em si do
>> problema, i.e., quais ternos (a*,b*,k*) são solução da equação.
>> Caso ninguém tenha resolvido a equação, ainda, gostaria como faço para
>> dar divulgação da minha conjectura, onde tenho a pretenção de ter
>> encontrado todas as soluções possíveis para a equação em epígrafe, no
>> Universo dos Naturais, com a restrição a>1, b>1 e K>1.
>>
>> Agradeço quem puder me orientar.
>>
>> Cordialmente,
>> PJMS
>>
>> --
>> Esta mensagem foi verificada pelo sistema de antivírus e
>> acredita-se estar livre de perigo.
>
>
> --
> Esta mensagem foi verificada pelo sistema de antivírus e
> acredita-se estar livre de perigo.

-- 
Esta mensagem foi verificada pelo sistema de antiv�rus e
 acredita-se estar livre de perigo.



Re: [obm-l] OBM 88 Problema 6.

2023-12-29 Por tôpico Claudio Buffara
Dá um Google em "IMO 88".
Vai ter até vídeo com a solução deste problema.

On Thu, Dec 28, 2023 at 4:35 PM Pedro José  wrote:

> Boa tarde!
> Com referência a esse problema criei uma conjectura, não consegui provar
> com a pretensão de abranger todas as soluções da equação:
>
> (a^2+b^2)/(ab+1)= k, com a,b,k Naturais e a>1, b>1 e k>1 Fiz essa
> restrição para retirar as soluções triviais.
> E SPG considerei a>b, já que a=b só ocorre para a=b=1, que está fora pela
> restrição acima e por ser uma equação simétrica em relação à a e b.
> O problema era provar que k era um quadrado perfeito.
> Gostaria de saber se alguém teria conhecimento da resolução em si do
> problema, i.e., quais ternos (a*,b*,k*) são solução da equação.
> Caso ninguém tenha resolvido a equação, ainda, gostaria como faço para dar
> divulgação da minha conjectura, onde tenho a pretenção de ter encontrado
> todas as soluções possíveis para a equação em epígrafe, no Universo dos
> Naturais, com a restrição a>1, b>1 e K>1.
>
> Agradeço quem puder me orientar.
>
> Cordialmente,
> PJMS
>
> --
> Esta mensagem foi verificada pelo sistema de antivírus e
> acredita-se estar livre de perigo.

-- 
Esta mensagem foi verificada pelo sistema de antiv�rus e
 acredita-se estar livre de perigo.



Re: [obm-l] OBM 88 Problema 6.

2023-12-28 Por tôpico Pedro José
Peço máxima vênia. Nem.reparata que fizera uma referência errada. OBM ao
invés de IMO. Interpretei erroneamente como uma censura. Só depois é que
reparei que falhará na referência.
Minhas escusas.

Cordialmente, PJMS.

Em qui., 28 de dez. de 2023 19:47, Anderson Torres <
torres.anderson...@gmail.com> escreveu:

>
>
> Em qui, 28 de dez de 2023 19:01, Pedro José 
> escreveu:
>
>> E daí?
>>
>
> E daí e daí?
>
>
>> Em qui., 28 de dez. de 2023 18:42, Anderson Torres <
>> torres.anderson...@gmail.com> escreveu:
>>
>>> Isso não é da OBM mas da IMO
>>>
>>> Em qui, 28 de dez de 2023 16:35, Pedro José 
>>> escreveu:
>>>
 Boa tarde!
 Com referência a esse problema criei uma conjectura, não consegui
 provar com a pretensão de abranger todas as soluções da equação:

 (a^2+b^2)/(ab+1)= k, com a,b,k Naturais e a>1, b>1 e k>1 Fiz essa
 restrição para retirar as soluções triviais.
 E SPG considerei a>b, já que a=b só ocorre para a=b=1, que está fora
 pela restrição acima e por ser uma equação simétrica em relação à a e b.
 O problema era provar que k era um quadrado perfeito.
 Gostaria de saber se alguém teria conhecimento da resolução em si do
 problema, i.e., quais ternos (a*,b*,k*) são solução da equação.

>>>
>>> Sim, o próprio método de resolução por descenso provê um método de
>>> listagem das soluções.
>>>
>>> Caso ninguém tenha resolvido a equação, ainda, gostaria como faço para
 dar divulgação da minha conjectura, onde tenho a pretenção de ter
 encontrado todas as soluções possíveis para a equação em epígrafe, no
 Universo dos Naturais, com a restrição a>1, b>1 e K>1.

 Agradeço quem puder me orientar.

 Cordialmente,
 PJMS

 --
 Esta mensagem foi verificada pelo sistema de antivírus e
 acredita-se estar livre de perigo.
>>>
>>>
>>> --
>>> Esta mensagem foi verificada pelo sistema de antivírus e
>>> acredita-se estar livre de perigo.
>>
>>
>> --
>> Esta mensagem foi verificada pelo sistema de antivírus e
>> acredita-se estar livre de perigo.
>
>
> --
> Esta mensagem foi verificada pelo sistema de antivírus e
> acredita-se estar livre de perigo.

-- 
Esta mensagem foi verificada pelo sistema de antiv�rus e
 acredita-se estar livre de perigo.



Re: [obm-l] OBM 88 Problema 6.

2023-12-28 Por tôpico Anderson Torres
Em qui, 28 de dez de 2023 19:01, Pedro José  escreveu:

> E daí?
>

E daí e daí?


> Em qui., 28 de dez. de 2023 18:42, Anderson Torres <
> torres.anderson...@gmail.com> escreveu:
>
>> Isso não é da OBM mas da IMO
>>
>> Em qui, 28 de dez de 2023 16:35, Pedro José 
>> escreveu:
>>
>>> Boa tarde!
>>> Com referência a esse problema criei uma conjectura, não consegui provar
>>> com a pretensão de abranger todas as soluções da equação:
>>>
>>> (a^2+b^2)/(ab+1)= k, com a,b,k Naturais e a>1, b>1 e k>1 Fiz essa
>>> restrição para retirar as soluções triviais.
>>> E SPG considerei a>b, já que a=b só ocorre para a=b=1, que está fora
>>> pela restrição acima e por ser uma equação simétrica em relação à a e b.
>>> O problema era provar que k era um quadrado perfeito.
>>> Gostaria de saber se alguém teria conhecimento da resolução em si do
>>> problema, i.e., quais ternos (a*,b*,k*) são solução da equação.
>>>
>>
>> Sim, o próprio método de resolução por descenso provê um método de
>> listagem das soluções.
>>
>> Caso ninguém tenha resolvido a equação, ainda, gostaria como faço para
>>> dar divulgação da minha conjectura, onde tenho a pretenção de ter
>>> encontrado todas as soluções possíveis para a equação em epígrafe, no
>>> Universo dos Naturais, com a restrição a>1, b>1 e K>1.
>>>
>>> Agradeço quem puder me orientar.
>>>
>>> Cordialmente,
>>> PJMS
>>>
>>> --
>>> Esta mensagem foi verificada pelo sistema de antivírus e
>>> acredita-se estar livre de perigo.
>>
>>
>> --
>> Esta mensagem foi verificada pelo sistema de antivírus e
>> acredita-se estar livre de perigo.
>
>
> --
> Esta mensagem foi verificada pelo sistema de antivírus e
> acredita-se estar livre de perigo.

-- 
Esta mensagem foi verificada pelo sistema de antiv�rus e
 acredita-se estar livre de perigo.



Re: [obm-l] OBM 88 Problema 6.

2023-12-28 Por tôpico Pedro José
E daí?

Em qui., 28 de dez. de 2023 18:42, Anderson Torres <
torres.anderson...@gmail.com> escreveu:

> Isso não é da OBM mas da IMO
>
> Em qui, 28 de dez de 2023 16:35, Pedro José 
> escreveu:
>
>> Boa tarde!
>> Com referência a esse problema criei uma conjectura, não consegui provar
>> com a pretensão de abranger todas as soluções da equação:
>>
>> (a^2+b^2)/(ab+1)= k, com a,b,k Naturais e a>1, b>1 e k>1 Fiz essa
>> restrição para retirar as soluções triviais.
>> E SPG considerei a>b, já que a=b só ocorre para a=b=1, que está fora pela
>> restrição acima e por ser uma equação simétrica em relação à a e b.
>> O problema era provar que k era um quadrado perfeito.
>> Gostaria de saber se alguém teria conhecimento da resolução em si do
>> problema, i.e., quais ternos (a*,b*,k*) são solução da equação.
>>
>
> Sim, o próprio método de resolução por descenso provê um método de
> listagem das soluções.
>
> Caso ninguém tenha resolvido a equação, ainda, gostaria como faço para dar
>> divulgação da minha conjectura, onde tenho a pretenção de ter encontrado
>> todas as soluções possíveis para a equação em epígrafe, no Universo dos
>> Naturais, com a restrição a>1, b>1 e K>1.
>>
>> Agradeço quem puder me orientar.
>>
>> Cordialmente,
>> PJMS
>>
>> --
>> Esta mensagem foi verificada pelo sistema de antivírus e
>> acredita-se estar livre de perigo.
>
>
> --
> Esta mensagem foi verificada pelo sistema de antivírus e
> acredita-se estar livre de perigo.

-- 
Esta mensagem foi verificada pelo sistema de antiv�rus e
 acredita-se estar livre de perigo.



Re: [obm-l] OBM 88 Problema 6.

2023-12-28 Por tôpico Anderson Torres
Isso não é da OBM mas da IMO

Em qui, 28 de dez de 2023 16:35, Pedro José  escreveu:

> Boa tarde!
> Com referência a esse problema criei uma conjectura, não consegui provar
> com a pretensão de abranger todas as soluções da equação:
>
> (a^2+b^2)/(ab+1)= k, com a,b,k Naturais e a>1, b>1 e k>1 Fiz essa
> restrição para retirar as soluções triviais.
> E SPG considerei a>b, já que a=b só ocorre para a=b=1, que está fora pela
> restrição acima e por ser uma equação simétrica em relação à a e b.
> O problema era provar que k era um quadrado perfeito.
> Gostaria de saber se alguém teria conhecimento da resolução em si do
> problema, i.e., quais ternos (a*,b*,k*) são solução da equação.
>

Sim, o próprio método de resolução por descenso provê um método de listagem
das soluções.

Caso ninguém tenha resolvido a equação, ainda, gostaria como faço para dar
> divulgação da minha conjectura, onde tenho a pretenção de ter encontrado
> todas as soluções possíveis para a equação em epígrafe, no Universo dos
> Naturais, com a restrição a>1, b>1 e K>1.
>
> Agradeço quem puder me orientar.
>
> Cordialmente,
> PJMS
>
> --
> Esta mensagem foi verificada pelo sistema de antivírus e
> acredita-se estar livre de perigo.

-- 
Esta mensagem foi verificada pelo sistema de antiv�rus e
 acredita-se estar livre de perigo.



Re: [obm-l] OBM 88 Problema 6.

2023-12-28 Por tôpico Anderson Torres
Em qui, 28 de dez de 2023 17:40, Bruno Bianchi Pagani <
brunobianchipag...@gmail.com> escreveu:

> Como que eu saio disso?
>

procure pelas instruções de unsubscribe.


> On Thu, Dec 28, 2023, 4:35 PM Pedro José  wrote:
>
>> Boa tarde!
>> Com referência a esse problema criei uma conjectura, não consegui provar
>> com a pretensão de abranger todas as soluções da equação:
>>
>> (a^2+b^2)/(ab+1)= k, com a,b,k Naturais e a>1, b>1 e k>1 Fiz essa
>> restrição para retirar as soluções triviais.
>> E SPG considerei a>b, já que a=b só ocorre para a=b=1, que está fora pela
>> restrição acima e por ser uma equação simétrica em relação à a e b.
>> O problema era provar que k era um quadrado perfeito.
>> Gostaria de saber se alguém teria conhecimento da resolução em si do
>> problema, i.e., quais ternos (a*,b*,k*) são solução da equação.
>> Caso ninguém tenha resolvido a equação, ainda, gostaria como faço para
>> dar divulgação da minha conjectura, onde tenho a pretenção de ter
>> encontrado todas as soluções possíveis para a equação em epígrafe, no
>> Universo dos Naturais, com a restrição a>1, b>1 e K>1.
>>
>> Agradeço quem puder me orientar.
>>
>> Cordialmente,
>> PJMS
>>
>> --
>> Esta mensagem foi verificada pelo sistema de antivírus e
>> acredita-se estar livre de perigo.
>
>
> --
> Esta mensagem foi verificada pelo sistema de antivírus e
> acredita-se estar livre de perigo.

-- 
Esta mensagem foi verificada pelo sistema de antiv�rus e
 acredita-se estar livre de perigo.



Re: [obm-l] OBM 88 Problema 6.

2023-12-28 Por tôpico Bruno Bianchi Pagani
Como que eu saio disso?

On Thu, Dec 28, 2023, 4:35 PM Pedro José  wrote:

> Boa tarde!
> Com referência a esse problema criei uma conjectura, não consegui provar
> com a pretensão de abranger todas as soluções da equação:
>
> (a^2+b^2)/(ab+1)= k, com a,b,k Naturais e a>1, b>1 e k>1 Fiz essa
> restrição para retirar as soluções triviais.
> E SPG considerei a>b, já que a=b só ocorre para a=b=1, que está fora pela
> restrição acima e por ser uma equação simétrica em relação à a e b.
> O problema era provar que k era um quadrado perfeito.
> Gostaria de saber se alguém teria conhecimento da resolução em si do
> problema, i.e., quais ternos (a*,b*,k*) são solução da equação.
> Caso ninguém tenha resolvido a equação, ainda, gostaria como faço para dar
> divulgação da minha conjectura, onde tenho a pretenção de ter encontrado
> todas as soluções possíveis para a equação em epígrafe, no Universo dos
> Naturais, com a restrição a>1, b>1 e K>1.
>
> Agradeço quem puder me orientar.
>
> Cordialmente,
> PJMS
>
> --
> Esta mensagem foi verificada pelo sistema de antivírus e
> acredita-se estar livre de perigo.

-- 
Esta mensagem foi verificada pelo sistema de antiv�rus e
 acredita-se estar livre de perigo.



[obm-l] OBM 88 Problema 6.

2023-12-28 Por tôpico Pedro José
Boa tarde!
Com referência a esse problema criei uma conjectura, não consegui provar
com a pretensão de abranger todas as soluções da equação:

(a^2+b^2)/(ab+1)= k, com a,b,k Naturais e a>1, b>1 e k>1 Fiz essa restrição
para retirar as soluções triviais.
E SPG considerei a>b, já que a=b só ocorre para a=b=1, que está fora pela
restrição acima e por ser uma equação simétrica em relação à a e b.
O problema era provar que k era um quadrado perfeito.
Gostaria de saber se alguém teria conhecimento da resolução em si do
problema, i.e., quais ternos (a*,b*,k*) são solução da equação.
Caso ninguém tenha resolvido a equação, ainda, gostaria como faço para dar
divulgação da minha conjectura, onde tenho a pretenção de ter encontrado
todas as soluções possíveis para a equação em epígrafe, no Universo dos
Naturais, com a restrição a>1, b>1 e K>1.

Agradeço quem puder me orientar.

Cordialmente,
PJMS

-- 
Esta mensagem foi verificada pelo sistema de antiv�rus e
 acredita-se estar livre de perigo.



Re: [obm-l] OBM 2022 Problema

2023-06-13 Por tôpico Felipe Giglio
problema 6 obm 2022 nivel 2

On Tue, Jun 13, 2023, 14:49 Anderson Torres 
wrote:

> De onde é esse problema?
>
> Em ter, 13 de jun de 2023 07:50, Prof. Douglas Oliveira <
> profdouglaso.del...@gmail.com> escreveu:
>
>> Determine o maior inteiro positivo k para o qual a afirmação é
>> verdadeira: Dados k subconjuntos distintos do conjunto {1, 2, 3, ...,
>> 2023}, cada um com 1011 elementos, é possível particionar os subconjuntos
>> em duas coleções em  a forma que quaisquer dois subconjuntos na mesma
>> coleção têm algum elemento em comum.
>>
>>
>>
>> Abraço do Douglas Oliveira.
>>
>> --
>> Esta mensagem foi verificada pelo sistema de antivírus e
>> acredita-se estar livre de perigo.
>
>
> --
> Esta mensagem foi verificada pelo sistema de antivírus e
> acredita-se estar livre de perigo.

-- 
Esta mensagem foi verificada pelo sistema de antiv�rus e
 acredita-se estar livre de perigo.



Re: [obm-l] OBM 2022 Problema

2023-06-13 Por tôpico Anderson Torres
De onde é esse problema?

Em ter, 13 de jun de 2023 07:50, Prof. Douglas Oliveira <
profdouglaso.del...@gmail.com> escreveu:

> Determine o maior inteiro positivo k para o qual a afirmação é verdadeira:
> Dados k subconjuntos distintos do conjunto {1, 2, 3, ..., 2023}, cada um
> com 1011 elementos, é possível particionar os subconjuntos em duas coleções
> em  a forma que quaisquer dois subconjuntos na mesma coleção têm algum
> elemento em comum.
>
>
>
> Abraço do Douglas Oliveira.
>
> --
> Esta mensagem foi verificada pelo sistema de antivírus e
> acredita-se estar livre de perigo.

-- 
Esta mensagem foi verificada pelo sistema de antiv�rus e
 acredita-se estar livre de perigo.



[obm-l] OBM 2022 Problema

2023-06-13 Por tôpico Prof. Douglas Oliveira
Determine o maior inteiro positivo k para o qual a afirmação é verdadeira:
Dados k subconjuntos distintos do conjunto {1, 2, 3, ..., 2023}, cada um
com 1011 elementos, é possível particionar os subconjuntos em duas coleções
em  a forma que quaisquer dois subconjuntos na mesma coleção têm algum
elemento em comum.



Abraço do Douglas Oliveira.

-- 
Esta mensagem foi verificada pelo sistema de antiv�rus e
 acredita-se estar livre de perigo.



[obm-l] Re: [obm-l] OBM e Olímpiadas internacionais

2022-12-11 Por tôpico Anderson Torres
Em qua., 7 de dez. de 2022 às 03:39, Obindinachukwu Desire Yema
 escreveu:
>
> Bom dia a todos,
> Nesse ano eu despertei um interesse em matemática pura, pensando um pouco 
> decidi que iria tentar no próximo ano fazer a OBM nivel universitário. 
> Pesquisando no site da OBM, eu não achei nada relacionado com o conteúdo que 
> cai na prova.
> Eu queria perguntar para vocês como que me preparo para a prova, no sentido 
> de: conteúdo que devo saber.
> Desde já agradeço a atenção.

De fato tem pouca coisa além das provas. Então, te sugiro pegar
pesado: estude a PUTNAM e a IMC. Com isso você vai ter mais material.

>
> --
> Esta mensagem foi verificada pelo sistema de antivírus e
> acredita-se estar livre de perigo.

-- 
Esta mensagem foi verificada pelo sistema de antiv�rus e
 acredita-se estar livre de perigo.


=
Instru��es para entrar na lista, sair da lista e usar a lista em
http://www.mat.puc-rio.br/~obmlistas/obm-l.html
=


[obm-l] Re: [obm-l] OBM e Olímpiadas internacionais

2022-12-07 Por tôpico Claudio Buffara
Eu começaria olhando as provas de anos anteriores, por exemplo aqui:
https://www.obm.org.br/como-se-preparar/provas-e-gabaritos/

On Wed, Dec 7, 2022 at 3:39 AM Obindinachukwu Desire Yema <
obindinachukwu.y...@usp.br> wrote:

> Bom dia a todos,
> Nesse ano eu despertei um interesse em matemática pura, pensando um pouco
> decidi que iria tentar no próximo ano fazer a OBM nivel universitário.
> Pesquisando no site da OBM, eu não achei nada relacionado com o conteúdo
> que cai na prova.
> Eu queria perguntar para vocês como que me preparo para a prova, no
> sentido de: conteúdo que devo saber.
> Desde já agradeço a atenção.
>
> --
> Esta mensagem foi verificada pelo sistema de antivírus e
> acredita-se estar livre de perigo.

-- 
Esta mensagem foi verificada pelo sistema de antiv�rus e
 acredita-se estar livre de perigo.



[obm-l] OBM e Olímpiadas internacionais

2022-12-06 Por tôpico Obindinachukwu Desire Yema
Bom dia a todos,
Nesse ano eu despertei um interesse em matemática pura, pensando um pouco
decidi que iria tentar no próximo ano fazer a OBM nivel universitário.
Pesquisando no site da OBM, eu não achei nada relacionado com o conteúdo
que cai na prova.
Eu queria perguntar para vocês como que me preparo para a prova, no sentido de:
conteúdo que devo saber.
Desde já agradeço a atenção.

-- 
Esta mensagem foi verificada pelo sistema de antiv�rus e
 acredita-se estar livre de perigo.



[obm-l] Re: [obm-l] [obm-l] Teoria dos Números

2020-10-07 Por tôpico Arthur Queiroz
Suponha que a =1. Queremos que 1/b + 1/c seja inteiro. Mas se b >= 3, temos
1/b + 1/c <= 2/3. Logo, as únicas sol nesse subcaso são b=c=1 e b=c=2.

Vou admitir como verdade que a<4 pq vc provou isso.

Suponha que 1 < a < 4 e b >= 5. Daí
1/a + 1/b + 1/c <= 1/2 + 1/5 + 1/5 = 9/10
Logo, b < 5. (Pq n há sol com b>=5 e a=1)

Suponha que a = 2. Daí 1/b + 1/c = 1/2. Se b>= 5, temos 1/b + 1/c <= 2/5.
Logo, b < 5. Testando as finitas possibilidades restantes, temos as sol
b=3, c=6 e b=4, c =4.

Suponha que 2 < a < 4, 2< b < 5 e c >= 4. Daí
1/a+1/b+1/c <= 1/3 + 1/3 + 1/4 < 1. Logo, c < 4. Mas se 2 >= c, teríamos 2
>= c >= b > 2, uma contradição. Logo, c = 3. E como c = 3 >= a, b > 2,
temos a = b = 3. Logo, a sol nesse subcaso seria a=3, b=3 e c=3

On Tue, Oct 6, 2020, 17:14 Marcos Duarte  wrote:

> Boa tarde!
>
> Encontre todos os números naturais a,b,c tais que a<=b<=c e a soma 1/a +
> 1/b + 1/c seja um inteiro.
>
> O único limitante que encontrei é que a < 4, pois 1/4 + 1/4 + 1/4 = 3/4 <
> 1 e já que a + 1 > a => 1/(a+1) < 1/a, temos que para a > 4 a soma continua
> menor que 1. Além disso, (1,1,1) e (3,3,3) satisfazem.
>
> --
> Esta mensagem foi verificada pelo sistema de antivírus e
> acredita-se estar livre de perigo.

-- 
Esta mensagem foi verificada pelo sistema de antiv�rus e
 acredita-se estar livre de perigo.



[obm-l] Re: [obm-l] [obm-l] Teoria dos Números

2020-10-06 Por tôpico Claudio Buffara
Há outros dois: (1,2,2) e (2,3,6).


On Tue, Oct 6, 2020 at 5:14 PM Marcos Duarte 
wrote:

> Boa tarde!
>
> Encontre todos os números naturais a,b,c tais que a<=b<=c e a soma 1/a +
> 1/b + 1/c seja um inteiro.
>
> O único limitante que encontrei é que a < 4, pois 1/4 + 1/4 + 1/4 = 3/4 <
> 1 e já que a + 1 > a => 1/(a+1) < 1/a, temos que para a > 4 a soma continua
> menor que 1. Além disso, (1,1,1) e (3,3,3) satisfazem.
>
> --
> Esta mensagem foi verificada pelo sistema de antivírus e
> acredita-se estar livre de perigo.

-- 
Esta mensagem foi verificada pelo sistema de antiv�rus e
 acredita-se estar livre de perigo.



[obm-l] [obm-l] Teoria dos Números

2020-10-06 Por tôpico Marcos Duarte
Boa tarde!

Encontre todos os números naturais a,b,c tais que a<=b<=c e a soma 1/a +
1/b + 1/c seja um inteiro.

O único limitante que encontrei é que a < 4, pois 1/4 + 1/4 + 1/4 = 3/4 < 1
e já que a + 1 > a => 1/(a+1) < 1/a, temos que para a > 4 a soma continua
menor que 1. Além disso, (1,1,1) e (3,3,3) satisfazem.

-- 
Esta mensagem foi verificada pelo sistema de antiv�rus e
 acredita-se estar livre de perigo.



Re: [obm-l] obm U

2020-02-22 Por tôpico Israel Meireles Chrisostomo
Muito obrigado pela resposta professor Douglas.
Quando vc diz cálculo e análise, vc inclui cálculo no R^n e análise no R^n?

Em sáb., 22 de fev. de 2020 às 13:28, Prof. Douglas Oliveira <
profdouglaso.del...@gmail.com> escreveu:

> Teoria dos números, combinatória, Geometria, análise, cálculo e álgebra.
>
> 
>
> Em sáb, 22 de fev de 2020 13:07, Israel Meireles Chrisostomo <
> israelmchrisost...@gmail.com> escreveu:
>
>>
>> Acho q eu não me fiz entender. Então eu quero saber só a matéria que cai
>> na obm nível U, tipo análise, álgebra, topologia, teoria dos números, etc
>>
>> O
>>
>> --
>> Israel Meireles Chrisostomo
>>
>> --
>> Esta mensagem foi verificada pelo sistema de antivírus e
>> acredita-se estar livre de perigo.
>
>
> --
> Esta mensagem foi verificada pelo sistema de antivírus e
> acredita-se estar livre de perigo.



-- 
Israel Meireles Chrisostomo

-- 
Esta mensagem foi verificada pelo sistema de antiv�rus e
 acredita-se estar livre de perigo.



Re: [obm-l] obm U

2020-02-22 Por tôpico Prof. Douglas Oliveira
Teoria dos números, combinatória, Geometria, análise, cálculo e álgebra.



Em sáb, 22 de fev de 2020 13:07, Israel Meireles Chrisostomo <
israelmchrisost...@gmail.com> escreveu:

>
> Acho q eu não me fiz entender. Então eu quero saber só a matéria que cai
> na obm nível U, tipo análise, álgebra, topologia, teoria dos números, etc
>
> O
>
> --
> Israel Meireles Chrisostomo
>
> --
> Esta mensagem foi verificada pelo sistema de antivírus e
> acredita-se estar livre de perigo.

-- 
Esta mensagem foi verificada pelo sistema de antiv�rus e
 acredita-se estar livre de perigo.



[obm-l] obm U

2020-02-22 Por tôpico Israel Meireles Chrisostomo
Acho q eu não me fiz entender. Então eu quero saber só a matéria que cai na
obm nível U, tipo análise, álgebra, topologia, teoria dos números, etc

O

-- 
Israel Meireles Chrisostomo

-- 
Esta mensagem foi verificada pelo sistema de antiv�rus e
 acredita-se estar livre de perigo.



[obm-l] OBM

2020-02-21 Por tôpico Israel Meireles Chrisostomo
Alguém aí tem todos os assuntos cobrados na OBM U e separado
 por tema?
Desde já agradeço
-- 
Israel Meireles Chrisostomo

-- 
Esta mensagem foi verificada pelo sistema de antiv�rus e
 acredita-se estar livre de perigo.



[obm-l] OBM Nível 1

2019-12-13 Por tôpico Mauricio Kanada
Olá a todos.
Alguém conseguiu fazer a questão 5-c da OBM 2019 Nível 1?
Eu cheguei ao resultado de 85 pontos, mas não de 84 pontos.

Grato
Maurício Kanada
-- 
Esta mensagem foi verificada pelo sistema de antiv�rus e
 acredita-se estar livre de perigo.



[obm-l] OBM 2019 Nível 1

2019-12-03 Por tôpico Mauricio Kanada
Olá a todos.
Não consegui resolver o problema 5, letra c da prova 2019 Nível 1 da OBM deste 
ano.
Consigo para o resultado de 85 pontos dentro do quadrado, mas não para 84 
pontos.
Alguma ajuda?
Alguém tem a resposta?
Grato
Maurício Kanada

-- 
Esta mensagem foi verificada pelo sistema de antiv�rus e
 acredita-se estar livre de perigo.



Re: [obm-l] Re: [obm-l] OBM nível u

2019-08-28 Por tôpico Wellington Jesus
Landim quando eu pergutava sobre essas bibliografias tu mandava eu estudar 
Física né!
 
 


Olá, Israel,
Primeiramente, irei comentar algumas outras bibliografias padrões:
Putnam and Beyond, do Tiru AndreescuBerkeley Problems in Mathematics, Ney de 
Souza
Algumas menos padrões para você treinar mais problem-solving são outros exames 
de admissão. Se você desenrola no francês, uma série muito boa é a seguinte:
exercices de mathématiques oraux x-ens, Francinou, Gianella & Nicolas
formada por problemas do exame oral de matemática para entrar na École 
Polytechnique e na École Normale Supérieure e diversas soluções.
Outros bons materiais são os da semana olímpica, disponíveis no site da OBM.
Ademais, existem diversas outras olimpíadas universitárias as quais você pode 
usar para treinar:
OIMU, CIIM, IMC, Galois-Noether, Vojtech Járnik.
Por fim, alguns professores disponibilizam sites com materiais focados em 
olimpíadas universitárias, os quais me lembro dois de cabeça.
Samuel Feitosa: 
https://sites.google.com/site/ufbasamuel/treinamento-obmuRicardo Bortolotti: 
https://www3.ufpe.br/rtbortolotti/index.php?option=com_content=article=362=240
Att.Thiago Landim.
On Wed, Aug 28, 2019, 19:07 Israel Meireles Chrisostomo 
 wrote:


Olá, alguém teria uma referencia bibliográfica para preparação para OBM U(além 
das provas anteriores)? Israel Meireles Chrisostomo
 
|  | Livre de vírus. www.avg.com.  |


--
Esta mensagem foi verificada pelo sistema de antivírus e 
 acredita-se estar livre de perigo.

--
Esta mensagem foi verificada pelo sistema de antiv�rus e 
 acredita-se estar livre de perigo.
-- 
Esta mensagem foi verificada pelo sistema de antiv�rus e
 acredita-se estar livre de perigo.



[obm-l] Re: [obm-l] Re: [obm-l] Re: [obm-l] OBM nível u

2019-08-28 Por tôpico Thiago Landim
Outras sugestões são os sites do
Yufei Zhao: http://yufeizhao.com/olympiad/
e do Evan Chen: http://web.evanchen.cc/recommend.html
além dos vários hiperlinks que eles citam, como por exemplo:
http://people.bath.ac.uk/masgcs/advice.html

Em qua, 28 de ago de 2019 às 20:51, Israel Meireles Chrisostomo <
israelmchrisost...@gmail.com> escreveu:

> muito obrigado Thiago!!!
>
>
> 
>  Livre
> de vírus. www.avg.com
> .
> <#m_2833610730167948638_DAB4FAD8-2DD7-40BB-A1B8-4E2AA1F9FDF2>
>
> Em qua, 28 de ago de 2019 às 19:39, Thiago Landim <
> thiago.landim...@gmail.com> escreveu:
>
>> Olá, Israel,
>>
>> Primeiramente, irei comentar algumas outras bibliografias padrões:
>>
>> Putnam and Beyond, do Tiru Andreescu
>> Berkeley Problems in Mathematics, Ney de Souza
>>
>> Algumas menos padrões para você treinar mais problem-solving são outros
>> exames de admissão. Se você desenrola no francês, uma série muito boa é a
>> seguinte:
>>
>> exercices de mathématiques oraux x-ens, Francinou, Gianella & Nicolas
>>
>> formada por problemas do exame oral de matemática para entrar na École
>> Polytechnique e na École Normale Supérieure e diversas soluções.
>>
>> Outros bons materiais são os da semana olímpica, disponíveis no site da
>> OBM.
>>
>> Ademais, existem diversas outras olimpíadas universitárias as quais você
>> pode usar para treinar:
>>
>> OIMU, CIIM, IMC, Galois-Noether, Vojtech Járnik.
>>
>> Por fim, alguns professores disponibilizam sites com materiais focados em
>> olimpíadas universitárias, os quais me lembro dois de cabeça.
>>
>> Samuel Feitosa: https://sites.google.com/site/ufbasamuel/treinamento-obmu
>> Ricardo Bortolotti:
>> https://www3.ufpe.br/rtbortolotti/index.php?option=com_content=article=362=240
>>
>> Att.
>> Thiago Landim.
>>
>> On Wed, Aug 28, 2019, 19:07 Israel Meireles Chrisostomo <
>> israelmchrisost...@gmail.com> wrote:
>>
>>>
>>> Olá, alguém teria uma referencia bibliográfica para preparação para OBM
>>> U(além das provas anteriores)?
>>> Israel Meireles Chrisostomo
>>>
>>>
>>> 
>>>  Livre
>>> de vírus. www.avg.com
>>> .
>>>
>>> <#m_2833610730167948638_m_6367409415193847683_m_-6854109204223493197_m_-220563535611758292_DAB4FAD8-2DD7-40BB-A1B8-4E2AA1F9FDF2>
>>>
>>> --
>>> Esta mensagem foi verificada pelo sistema de antivírus e
>>> acredita-se estar livre de perigo.
>>
>>
>> --
>> Esta mensagem foi verificada pelo sistema de antivírus e
>> acredita-se estar livre de perigo.
>
>
>
> --
> Israel Meireles Chrisostomo
>
> --
> Esta mensagem foi verificada pelo sistema de antivírus e
> acredita-se estar livre de perigo.

-- 
Esta mensagem foi verificada pelo sistema de antiv�rus e
 acredita-se estar livre de perigo.



[obm-l] Re: [obm-l] Re: [obm-l] OBM nível u

2019-08-28 Por tôpico Israel Meireles Chrisostomo
muito obrigado Thiago!!!


Livre
de vírus. www.avg.com
.
<#DAB4FAD8-2DD7-40BB-A1B8-4E2AA1F9FDF2>

Em qua, 28 de ago de 2019 às 19:39, Thiago Landim <
thiago.landim...@gmail.com> escreveu:

> Olá, Israel,
>
> Primeiramente, irei comentar algumas outras bibliografias padrões:
>
> Putnam and Beyond, do Tiru Andreescu
> Berkeley Problems in Mathematics, Ney de Souza
>
> Algumas menos padrões para você treinar mais problem-solving são outros
> exames de admissão. Se você desenrola no francês, uma série muito boa é a
> seguinte:
>
> exercices de mathématiques oraux x-ens, Francinou, Gianella & Nicolas
>
> formada por problemas do exame oral de matemática para entrar na École
> Polytechnique e na École Normale Supérieure e diversas soluções.
>
> Outros bons materiais são os da semana olímpica, disponíveis no site da
> OBM.
>
> Ademais, existem diversas outras olimpíadas universitárias as quais você
> pode usar para treinar:
>
> OIMU, CIIM, IMC, Galois-Noether, Vojtech Járnik.
>
> Por fim, alguns professores disponibilizam sites com materiais focados em
> olimpíadas universitárias, os quais me lembro dois de cabeça.
>
> Samuel Feitosa: https://sites.google.com/site/ufbasamuel/treinamento-obmu
> Ricardo Bortolotti:
> https://www3.ufpe.br/rtbortolotti/index.php?option=com_content=article=362=240
>
> Att.
> Thiago Landim.
>
> On Wed, Aug 28, 2019, 19:07 Israel Meireles Chrisostomo <
> israelmchrisost...@gmail.com> wrote:
>
>>
>> Olá, alguém teria uma referencia bibliográfica para preparação para OBM
>> U(além das provas anteriores)?
>> Israel Meireles Chrisostomo
>>
>>
>> 
>>  Livre
>> de vírus. www.avg.com
>> .
>>
>> <#m_6367409415193847683_m_-6854109204223493197_m_-220563535611758292_DAB4FAD8-2DD7-40BB-A1B8-4E2AA1F9FDF2>
>>
>> --
>> Esta mensagem foi verificada pelo sistema de antivírus e
>> acredita-se estar livre de perigo.
>
>
> --
> Esta mensagem foi verificada pelo sistema de antivírus e
> acredita-se estar livre de perigo.



-- 
Israel Meireles Chrisostomo

-- 
Esta mensagem foi verificada pelo sistema de antiv�rus e
 acredita-se estar livre de perigo.



[obm-l] Re: [obm-l] OBM nível u

2019-08-28 Por tôpico Thiago Landim
Olá, Israel,

Primeiramente, irei comentar algumas outras bibliografias padrões:

Putnam and Beyond, do Tiru Andreescu
Berkeley Problems in Mathematics, Ney de Souza

Algumas menos padrões para você treinar mais problem-solving são outros
exames de admissão. Se você desenrola no francês, uma série muito boa é a
seguinte:

exercices de mathématiques oraux x-ens, Francinou, Gianella & Nicolas

formada por problemas do exame oral de matemática para entrar na École
Polytechnique e na École Normale Supérieure e diversas soluções.

Outros bons materiais são os da semana olímpica, disponíveis no site da OBM.

Ademais, existem diversas outras olimpíadas universitárias as quais você
pode usar para treinar:

OIMU, CIIM, IMC, Galois-Noether, Vojtech Járnik.

Por fim, alguns professores disponibilizam sites com materiais focados em
olimpíadas universitárias, os quais me lembro dois de cabeça.

Samuel Feitosa: https://sites.google.com/site/ufbasamuel/treinamento-obmu
Ricardo Bortolotti:
https://www3.ufpe.br/rtbortolotti/index.php?option=com_content=article=362=240

Att.
Thiago Landim.

On Wed, Aug 28, 2019, 19:07 Israel Meireles Chrisostomo <
israelmchrisost...@gmail.com> wrote:

>
> Olá, alguém teria uma referencia bibliográfica para preparação para OBM
> U(além das provas anteriores)?
> Israel Meireles Chrisostomo
>
>
> 
>  Livre
> de vírus. www.avg.com
> .
>
> <#m_-6854109204223493197_m_-220563535611758292_DAB4FAD8-2DD7-40BB-A1B8-4E2AA1F9FDF2>
>
> --
> Esta mensagem foi verificada pelo sistema de antivírus e
> acredita-se estar livre de perigo.

-- 
Esta mensagem foi verificada pelo sistema de antiv�rus e
 acredita-se estar livre de perigo.



[obm-l] OBM nível u

2019-08-28 Por tôpico Israel Meireles Chrisostomo
Olá, alguém teria uma referencia bibliográfica para preparação para OBM
U(além das provas anteriores)?
Israel Meireles Chrisostomo


Livre
de vírus. www.avg.com
.
<#DAB4FAD8-2DD7-40BB-A1B8-4E2AA1F9FDF2>

-- 
Esta mensagem foi verificada pelo sistema de antiv�rus e
 acredita-se estar livre de perigo.



[obm-l] Re: [obm-l] Re: [obm-l] Obm Nível 2 2017

2019-04-05 Por tôpico Gabriel Lopes
Obrigado, achei meio nebuloso mas vou tentar entender " Então tiraremos n^2
cores idênticas a iniciais e (n-1) as cores da primeira coluna " esse
processo nao consegui entender , tirar n^2 cores e nao ter cor alguma
nao? "(n-1)
conjuntos iguais ao iniciais." E aqui nao seriam n conjuntos? Logo depois
vc fala 2n-1 conjuntos identicos, desculpa mas eu nao consegui
compreender.vou me esforçar para absorver.

Tinha pensando no seguinte , tentar provar  q em um tabuleiro n par, se as
colunas  e linhas tivesse o msm numero de elementos entao a malha seria
xadrez, quem sabe por indução, mas seria uma indução para ordem pares
somente, mas nao ta saindo 

Em Sex, 5 de abr de 2019 17:23, Pedro José  escreveu:

> Boa tarde!
> Caso n seja par está resolvido. Pois, sobrará uma quantidade ímpar de
> casas e portanto não há como serem iguais em quantidade.
>
> Caso n ímpar. Uma das cores prevalecerá. Suponhamos que tenhamos X de uma
> cor e X + k da outra com 2X+k=n^2 e k>0
> Nós temos n^2 formas de tirar uma linha e uma coluna.
> Cada vez que tiramos uma linha e uma coluna, tiramos 2n-1 casas. Para que
> fique igual temos que tirar x da que tem menor quantidade e x+K da que tem
> mais, e 2x+k = 2n-1.
> Vamos fazer a seguinte varredura.
> Para cada coluna vamos varrer todas as n linhasEntão tiraremos n^2 cores
> idênticas a iniciais e (n-1) as cores da primeira coluna , depois n^2
> cores idênticas a iniciais e (n-1) as cores da segunda coluna... Ao final
> tiraremos n conjuntos de cores iguais as n^2 iniciais + (n-1) conjuntos
> iguais ao iniciais.
> Ou seja: (2n-1) conjuntos idênticos ao inicial. O que acarreta em (2n-1) X
> de uma cor e (2n-1) (X+k) de outra apresentando uma diferença de (2n-1)K.
> Para que em todas essas retiradas (uma linha e uma coluna) sobrem cores
> idênticas é necessário se retirar de cada vez x da cor em menor quantidade
> e x + k da maior, onde
> 2x+k = 2n-1, todas as vezes. Há n^2 possibilidades de tirar uma linha e
> uma coluna portanto serão retiradas n^2(2n-1), como já visto, só que n^2*
> (x+k) e n^2(x), o que dá uma diferença de n^2k. Mas pelo outro método dava
> (2n-1)k ==> n^2 =2n-1 ==> n= 1  absurdo, pois n>1.
> Portanto, em alguma retirada sobrarão mais de uma cor que de outra.
>
> Saudações,
> PJMS.
>
> Em qui, 4 de abr de 2019 às 01:06, Gabriel Lopes 
> escreveu:
>
>> Seja *n>1* um inteiro e considere um tabuleiro *nxn*, em que algumas das
>> *n²* casas foram pintadas de pretos, e as restantes foram pintadas de
>> branco. Prove que é possível escolhermos uma das *n²* casas do
>> tabuleiro, de modo que, ao removermos completamente a linha e a coluna que
>> a contém, haja um número diferente de casas pretas e de casas brancas,
>> dentre as *(n-1)².*
>>
>> --
>> Esta mensagem foi verificada pelo sistema de antivírus e
>> acredita-se estar livre de perigo.
>
>
> --
> Esta mensagem foi verificada pelo sistema de antivírus e
> acredita-se estar livre de perigo.

-- 
Esta mensagem foi verificada pelo sistema de antiv�rus e
 acredita-se estar livre de perigo.



[obm-l] Re: [obm-l] Obm Nível 2 2017

2019-04-05 Por tôpico Pedro José
Boa tarde!
Caso n seja par está resolvido. Pois, sobrará uma quantidade ímpar de casas
e portanto não há como serem iguais em quantidade.

Caso n ímpar. Uma das cores prevalecerá. Suponhamos que tenhamos X de uma
cor e X + k da outra com 2X+k=n^2 e k>0
Nós temos n^2 formas de tirar uma linha e uma coluna.
Cada vez que tiramos uma linha e uma coluna, tiramos 2n-1 casas. Para que
fique igual temos que tirar x da que tem menor quantidade e x+K da que tem
mais, e 2x+k = 2n-1.
Vamos fazer a seguinte varredura.
Para cada coluna vamos varrer todas as n linhas. Então tiraremos n^2 cores
idênticas a iniciais e (n-1) as cores da primeira coluna, depois n^2 cores
idênticas a iniciais e (n-1) as cores da segunda coluna... Ao final
tiraremos n conjuntos de cores iguais as n^2 iniciais + (n-1) conjuntos
iguais ao iniciais.
Ou seja: (2n-1) conjuntos idênticos ao inicial. O que acarreta em (2n-1) X
de uma cor e (2n-1) (X+k) de outra apresentando uma diferença de (2n-1)K.
Para que em todas essas retiradas (uma linha e uma coluna) sobrem cores
idênticas é necessário se retirar de cada vez x da cor em menor quantidade
e x + k da maior, onde
2x+k = 2n-1, todas as vezes. Há n^2 possibilidades de tirar uma linha e uma
coluna portanto serão retiradas n^2(2n-1), como já visto, só que n^2* (x+k)
e n^2(x), o que dá uma diferença de n^2k. Mas pelo outro método dava
(2n-1)k ==> n^2 =2n-1 ==> n= 1  absurdo, pois n>1.
Portanto, em alguma retirada sobrarão mais de uma cor que de outra.

Saudações,
PJMS.

Em qui, 4 de abr de 2019 às 01:06, Gabriel Lopes 
escreveu:

> Seja *n>1* um inteiro e considere um tabuleiro *nxn*, em que algumas das
> *n²* casas foram pintadas de pretos, e as restantes foram pintadas de
> branco. Prove que é possível escolhermos uma das *n²* casas do tabuleiro,
> de modo que, ao removermos completamente a linha e a coluna que a contém,
> haja um número diferente de casas pretas e de casas brancas, dentre as
> *(n-1)².*
>
> --
> Esta mensagem foi verificada pelo sistema de antivírus e
> acredita-se estar livre de perigo.

-- 
Esta mensagem foi verificada pelo sistema de antiv�rus e
 acredita-se estar livre de perigo.



[obm-l] Obm Nível 2 2017

2019-04-03 Por tôpico Gabriel Lopes
Seja *n>1* um inteiro e considere um tabuleiro *nxn*, em que algumas das
*n²* casas foram pintadas de pretos, e as restantes foram pintadas de
branco. Prove que é possível escolhermos uma das *n²* casas do tabuleiro,
de modo que, ao removermos completamente a linha e a coluna que a contém,
haja um número diferente de casas pretas e de casas brancas, dentre as
*(n-1)².*

-- 
Esta mensagem foi verificada pelo sistema de antiv�rus e
 acredita-se estar livre de perigo.



[obm-l] Re: [obm-l] OBM 2018 - Nível 1 - Questão 2

2019-03-26 Por tôpico Pedro José
Boa noite!

Não mencionei que embora no braço não seja um trabalho hercúleo, pois, o
resto por 9, se obtém com somas consecutivas dos algarismos.

1, 2, 2, 4, 8, 5 (3+2), 4 (4+0), 2 ,8, 7, 2, 5, 1, 5, 5, 7, 8, 2, 7, 5, 8,
4, 5, 2, 1, 2... Pronto achado o período 24.

De toda sorte deve ter forma mais fácil.

Saudações,
PJMS


Em ter, 26 de mar de 2019 às 19:11, Pedro José 
escreveu:

> Boa noite!
>
> a) O quarto termo é nulo e a partir daí todos também são.
> b) Esse, deve ter uma solução mais elegante. Fiz no braço e dá um período
> de 24. Logo dá o mesmo termo da ordem do resto de 2018 por 24 que é 2.
> Portanto, dá o segundo termo que por coincidência é 2.
> c) 1,1,1,1,1,1,1,1,1,1,1,1,1,1,1,1 e
> 1,10,1,1,1,1,1,1,1,1,1,1,1,1,1.
>
> Quanto ao b deve ter um modo de encontrar o período, sem ser no braço.
>
> Saudações,
> PJMS.
>
> Em ter, 26 de mar de 2019 às 18:24,  escreveu:
>
>> 2. Considere abaixo as sequências de números inteiros que possuem as duas
>> propriedades a seguir:
>>
>> i) Os dois primeiros termos são dados.
>> ii) Cada um dos termos seguintes e o resto da divisão por 9 do produto
>> dos dois termos anteriores.
>>
>> a) Qual e o vigésimo termo da sequência: 2, 3, ...?
>> b) Qual e o 2018 o termo da sequencia 1, 2, ...?
>> c) Apresente duas sequências cujo 2018 o termo e igual a 1.
>>
>>
>>
>> --
>> Esta mensagem foi verificada pelo sistema de antivírus e
>> acredita-se estar livre de perigo.
>>
>

-- 
Esta mensagem foi verificada pelo sistema de antiv�rus e
 acredita-se estar livre de perigo.



[obm-l] Re: [obm-l] OBM 2018 - Nível 1 - Questão 2

2019-03-26 Por tôpico Pedro José
Boa noite!

a) O quarto termo é nulo e a partir daí todos também são.
b) Esse, deve ter uma solução mais elegante. Fiz no braço e dá um período
de 24. Logo dá o mesmo termo da ordem do resto de 2018 por 24 que é 2.
Portanto, dá o segundo termo que por coincidência é 2.
c) 1,1,1,1,1,1,1,1,1,1,1,1,1,1,1,1 e 1,10,1,1,1,1,1,1,1,1,1,1,1,1,1.

Quanto ao b deve ter um modo de encontrar o período, sem ser no braço.

Saudações,
PJMS.

Em ter, 26 de mar de 2019 às 18:24,  escreveu:

> 2. Considere abaixo as sequências de números inteiros que possuem as duas
> propriedades a seguir:
>
> i) Os dois primeiros termos são dados.
> ii) Cada um dos termos seguintes e o resto da divisão por 9 do produto dos
> dois termos anteriores.
>
> a) Qual e o vigésimo termo da sequência: 2, 3, ...?
> b) Qual e o 2018 o termo da sequencia 1, 2, ...?
> c) Apresente duas sequências cujo 2018 o termo e igual a 1.
>
>
>
> --
> Esta mensagem foi verificada pelo sistema de antivírus e
> acredita-se estar livre de perigo.
>

-- 
Esta mensagem foi verificada pelo sistema de antiv�rus e
 acredita-se estar livre de perigo.



[obm-l] OBM 2018 - Nível 1 - Questão 2

2019-03-26 Por tôpico jjunior
2. Considere abaixo as sequências de números inteiros que possuem as
duas propriedades a seguir: 

i) Os dois primeiros termos são dados. 
ii) Cada um dos termos seguintes e o resto da divisão por 9 do
produto dos dois termos anteriores. 

a) Qual e o vigésimo termo da sequência: 2, 3, ...? 
b) Qual e o 2018 o termo da sequencia 1, 2, ...? 
c) Apresente duas sequências cujo 2018 o termo e igual a 1.



-- 
Esta mensagem foi verificada pelo sistema de antiv�rus e
 acredita-se estar livre de perigo.



Re: [obm-l] OBM 2002 - Problema 6

2019-01-13 Por tôpico Pedro José
Bruno,
 realmente eu viajei. As palavras nao podem ter mais de 16 caracteres
iquais.

Saudações,
PJMS



Em dom, 13 de jan de 2019 18:28, Bruno Visnadi  Me parece que o erro está na primeira premissa de que não podemos repetir
> as 8 primeiras posições.
> A condição do problema é que qualquer par de palavras se difira em 8
> posições. Isto é, eles podem ser iguais em até 16 posições.
>
> Em Dom, 13 de jan de 2019 18:11, Pedro José 
>> Boa tarde!
>> Suponho ter achado uma solução. Mas pela simplicidade, receio estar
>> errada. Fica para ser descartada ou corroborada.
>> 1) Vamos primeiro propor que não repitamos as 8 primeiras posições.
>> Fixando-se os balores das primeiras 8 posições, tenho 2^16 sequencis de
>> 24 posições. Das quais só posso aproveitar 1.
>> Portanto tenho de descartar 2^16-1 sequencias.
>> 2) Como tenho 2^8 possipilidades de escolher as 8 primeiras posições,
>> para nunca repertir as 8 primeiras, tenho que descartar ao total:
>> 2^8*(2^16-1)=2^24-2^8
>> Como tenho 2^24 sequências ao total, só sobrariam 2^8 <4096.
>> Como utilizei uma restrição mais branda, para o proposto será menor ainda
>> o número de possibilidafes.
>> Será aue está correto?
>> Saudações,
>> PJMS
>>
>>
>>
>> Em dom, 13 de jan de 2019 15:37, Pedro José > escreveu:
>>
>>> Jéferson,
>>> a sugestão do Cláudio é um caminho.
>>> Mas me perdoem-me pela intromissão. Parece que você não percebeu que é
>>> um problema de contagem. Você tem 24 casas para preencher com G ou P, mas
>>> não pode haver em nenhuma escolha 8 posições preenchidas com os mesmos
>>> valores.
>>>
>>> Em dom, 13 de jan de 2019 14:13, Claudio Buffara <
>>> claudio.buff...@gmail.com escreveu:
>>>
 Tente fazer casos menores, digamos de comprimento 6 ou 8 e diferindo em
 pelo menos 2 ou 4 posições.
 Deve dar pra fazer na mão (enumeração direta e braçal) e talvez permita
 detectar alguma lei de formação.

 On Sat, Jan 12, 2019 at 10:23 PM Jeferson Almir <
 jefersonram...@gmail.com> wrote:

> Amigos peço ajuda nesse problema, e me orientaram a estudar Códigos
> Corretores de Erros.
>
>
> Arnaldo e Beatriz se comunicam durante um acampamento usando sinais de
> fumaça, às vezes usando uma nuvem grande, às vezes uma pequena.
>
> No tempo disponível antes do café da manhã, Arnaldo consegue enviar
> uma seqüência de 24 nuvens. Como Beatriz nem sempre consegue distinguir 
> uma
> nuvem pequena de uma grande, ela e Arnaldo fizeram um dicionário antes de
> ir para o acampamento. No dicionário aparecem N seqüências de 24 tamanhos
> de nuvem (como por exemplo a seqüência PGPGPGPGPGPGGPGPGPGPGPGP, onde G
> significa nuvem grande e P significa nuvem pequena). Para cada uma das N
> seqüências, o dicionário indica seu significado. Para evitar 
> interpretações
> erradas, Arnaldo e Beatriz evitaram incluir no dicionário seqüências
> parecidas. Mais precisamente, duas seqüências no dicionário sempre diferem
> em pelo menos 8 das 24 posições.
>
>
>
> Demonstre que N<= 4096
>
> --
> Esta mensagem foi verificada pelo sistema de antivírus e
> acredita-se estar livre de perigo.


 --
 Esta mensagem foi verificada pelo sistema de antivírus e
 acredita-se estar livre de perigo.
>>>
>>>
>> --
>> Esta mensagem foi verificada pelo sistema de antivírus e
>> acredita-se estar livre de perigo.
>
>
> --
> Esta mensagem foi verificada pelo sistema de antivírus e
> acredita-se estar livre de perigo.

-- 
Esta mensagem foi verificada pelo sistema de antiv�rus e
 acredita-se estar livre de perigo.



Re: [obm-l] OBM 2002 - Problema 6

2019-01-13 Por tôpico Bruno Visnadi
Me parece que o erro está na primeira premissa de que não podemos repetir
as 8 primeiras posições.
A condição do problema é que qualquer par de palavras se difira em 8
posições. Isto é, eles podem ser iguais em até 16 posições.

Em Dom, 13 de jan de 2019 18:11, Pedro José  Boa tarde!
> Suponho ter achado uma solução. Mas pela simplicidade, receio estar
> errada. Fica para ser descartada ou corroborada.
> 1) Vamos primeiro propor que não repitamos as 8 primeiras posições.
> Fixando-se os balores das primeiras 8 posições, tenho 2^16 sequencis de 24
> posições. Das quais só posso aproveitar 1.
> Portanto tenho de descartar 2^16-1 sequencias.
> 2) Como tenho 2^8 possipilidades de escolher as 8 primeiras posições, para
> nunca repertir as 8 primeiras, tenho que descartar ao total:
> 2^8*(2^16-1)=2^24-2^8
> Como tenho 2^24 sequências ao total, só sobrariam 2^8 <4096.
> Como utilizei uma restrição mais branda, para o proposto será menor ainda
> o número de possibilidafes.
> Será aue está correto?
> Saudações,
> PJMS
>
>
>
> Em dom, 13 de jan de 2019 15:37, Pedro José 
>> Jéferson,
>> a sugestão do Cláudio é um caminho.
>> Mas me perdoem-me pela intromissão. Parece que você não percebeu que é um
>> problema de contagem. Você tem 24 casas para preencher com G ou P, mas não
>> pode haver em nenhuma escolha 8 posições preenchidas com os mesmos valores.
>>
>> Em dom, 13 de jan de 2019 14:13, Claudio Buffara <
>> claudio.buff...@gmail.com escreveu:
>>
>>> Tente fazer casos menores, digamos de comprimento 6 ou 8 e diferindo em
>>> pelo menos 2 ou 4 posições.
>>> Deve dar pra fazer na mão (enumeração direta e braçal) e talvez permita
>>> detectar alguma lei de formação.
>>>
>>> On Sat, Jan 12, 2019 at 10:23 PM Jeferson Almir <
>>> jefersonram...@gmail.com> wrote:
>>>
 Amigos peço ajuda nesse problema, e me orientaram a estudar Códigos
 Corretores de Erros.


 Arnaldo e Beatriz se comunicam durante um acampamento usando sinais de
 fumaça, às vezes usando uma nuvem grande, às vezes uma pequena.

 No tempo disponível antes do café da manhã, Arnaldo consegue enviar uma
 seqüência de 24 nuvens. Como Beatriz nem sempre consegue distinguir uma
 nuvem pequena de uma grande, ela e Arnaldo fizeram um dicionário antes de
 ir para o acampamento. No dicionário aparecem N seqüências de 24 tamanhos
 de nuvem (como por exemplo a seqüência PGPGPGPGPGPGGPGPGPGPGPGP, onde G
 significa nuvem grande e P significa nuvem pequena). Para cada uma das N
 seqüências, o dicionário indica seu significado. Para evitar interpretações
 erradas, Arnaldo e Beatriz evitaram incluir no dicionário seqüências
 parecidas. Mais precisamente, duas seqüências no dicionário sempre diferem
 em pelo menos 8 das 24 posições.



 Demonstre que N<= 4096

 --
 Esta mensagem foi verificada pelo sistema de antivírus e
 acredita-se estar livre de perigo.
>>>
>>>
>>> --
>>> Esta mensagem foi verificada pelo sistema de antivírus e
>>> acredita-se estar livre de perigo.
>>
>>
> --
> Esta mensagem foi verificada pelo sistema de antivírus e
> acredita-se estar livre de perigo.

-- 
Esta mensagem foi verificada pelo sistema de antiv�rus e
 acredita-se estar livre de perigo.



Re: [obm-l] OBM 2002 - Problema 6

2019-01-13 Por tôpico Pedro José
Boa tarde!
Suponho ter achado uma solução. Mas pela simplicidade, receio estar errada.
Fica para ser descartada ou corroborada.
1) Vamos primeiro propor que não repitamos as 8 primeiras posições.
Fixando-se os balores das primeiras 8 posições, tenho 2^16 sequencis de 24
posições. Das quais só posso aproveitar 1.
Portanto tenho de descartar 2^16-1 sequencias.
2) Como tenho 2^8 possipilidades de escolher as 8 primeiras posições, para
nunca repertir as 8 primeiras, tenho que descartar ao total:
2^8*(2^16-1)=2^24-2^8
Como tenho 2^24 sequências ao total, só sobrariam 2^8 <4096.
Como utilizei uma restrição mais branda, para o proposto será menor ainda o
número de possibilidafes.
Será aue está correto?
Saudações,
PJMS



Em dom, 13 de jan de 2019 15:37, Pedro José  Jéferson,
> a sugestão do Cláudio é um caminho.
> Mas me perdoem-me pela intromissão. Parece que você não percebeu que é um
> problema de contagem. Você tem 24 casas para preencher com G ou P, mas não
> pode haver em nenhuma escolha 8 posições preenchidas com os mesmos valores.
>
> Em dom, 13 de jan de 2019 14:13, Claudio Buffara <
> claudio.buff...@gmail.com escreveu:
>
>> Tente fazer casos menores, digamos de comprimento 6 ou 8 e diferindo em
>> pelo menos 2 ou 4 posições.
>> Deve dar pra fazer na mão (enumeração direta e braçal) e talvez permita
>> detectar alguma lei de formação.
>>
>> On Sat, Jan 12, 2019 at 10:23 PM Jeferson Almir 
>> wrote:
>>
>>> Amigos peço ajuda nesse problema, e me orientaram a estudar Códigos
>>> Corretores de Erros.
>>>
>>>
>>> Arnaldo e Beatriz se comunicam durante um acampamento usando sinais de
>>> fumaça, às vezes usando uma nuvem grande, às vezes uma pequena.
>>>
>>> No tempo disponível antes do café da manhã, Arnaldo consegue enviar uma
>>> seqüência de 24 nuvens. Como Beatriz nem sempre consegue distinguir uma
>>> nuvem pequena de uma grande, ela e Arnaldo fizeram um dicionário antes de
>>> ir para o acampamento. No dicionário aparecem N seqüências de 24 tamanhos
>>> de nuvem (como por exemplo a seqüência PGPGPGPGPGPGGPGPGPGPGPGP, onde G
>>> significa nuvem grande e P significa nuvem pequena). Para cada uma das N
>>> seqüências, o dicionário indica seu significado. Para evitar interpretações
>>> erradas, Arnaldo e Beatriz evitaram incluir no dicionário seqüências
>>> parecidas. Mais precisamente, duas seqüências no dicionário sempre diferem
>>> em pelo menos 8 das 24 posições.
>>>
>>>
>>>
>>> Demonstre que N<= 4096
>>>
>>> --
>>> Esta mensagem foi verificada pelo sistema de antivírus e
>>> acredita-se estar livre de perigo.
>>
>>
>> --
>> Esta mensagem foi verificada pelo sistema de antivírus e
>> acredita-se estar livre de perigo.
>
>

-- 
Esta mensagem foi verificada pelo sistema de antiv�rus e
 acredita-se estar livre de perigo.



Re: [obm-l] OBM 2002 - Problema 6

2019-01-13 Por tôpico Pedro José
Jéferson,
a sugestão do Cláudio é um caminho.
Mas me perdoem-me pela intromissão. Parece que você não percebeu que é um
problema de contagem. Você tem 24 casas para preencher com G ou P, mas não
pode haver em nenhuma escolha 8 posições preenchidas com os mesmos valores.

Em dom, 13 de jan de 2019 14:13, Claudio Buffara  Tente fazer casos menores, digamos de comprimento 6 ou 8 e diferindo em
> pelo menos 2 ou 4 posições.
> Deve dar pra fazer na mão (enumeração direta e braçal) e talvez permita
> detectar alguma lei de formação.
>
> On Sat, Jan 12, 2019 at 10:23 PM Jeferson Almir 
> wrote:
>
>> Amigos peço ajuda nesse problema, e me orientaram a estudar Códigos
>> Corretores de Erros.
>>
>>
>> Arnaldo e Beatriz se comunicam durante um acampamento usando sinais de
>> fumaça, às vezes usando uma nuvem grande, às vezes uma pequena.
>>
>> No tempo disponível antes do café da manhã, Arnaldo consegue enviar uma
>> seqüência de 24 nuvens. Como Beatriz nem sempre consegue distinguir uma
>> nuvem pequena de uma grande, ela e Arnaldo fizeram um dicionário antes de
>> ir para o acampamento. No dicionário aparecem N seqüências de 24 tamanhos
>> de nuvem (como por exemplo a seqüência PGPGPGPGPGPGGPGPGPGPGPGP, onde G
>> significa nuvem grande e P significa nuvem pequena). Para cada uma das N
>> seqüências, o dicionário indica seu significado. Para evitar interpretações
>> erradas, Arnaldo e Beatriz evitaram incluir no dicionário seqüências
>> parecidas. Mais precisamente, duas seqüências no dicionário sempre diferem
>> em pelo menos 8 das 24 posições.
>>
>>
>>
>> Demonstre que N<= 4096
>>
>> --
>> Esta mensagem foi verificada pelo sistema de antivírus e
>> acredita-se estar livre de perigo.
>
>
> --
> Esta mensagem foi verificada pelo sistema de antivírus e
> acredita-se estar livre de perigo.

-- 
Esta mensagem foi verificada pelo sistema de antiv�rus e
 acredita-se estar livre de perigo.



Re: [obm-l] OBM 2002 - Problema 6

2019-01-13 Por tôpico Claudio Buffara
Tente fazer casos menores, digamos de comprimento 6 ou 8 e diferindo em
pelo menos 2 ou 4 posições.
Deve dar pra fazer na mão (enumeração direta e braçal) e talvez permita
detectar alguma lei de formação.

On Sat, Jan 12, 2019 at 10:23 PM Jeferson Almir 
wrote:

> Amigos peço ajuda nesse problema, e me orientaram a estudar Códigos
> Corretores de Erros.
>
>
> Arnaldo e Beatriz se comunicam durante um acampamento usando sinais de
> fumaça, às vezes usando uma nuvem grande, às vezes uma pequena.
>
> No tempo disponível antes do café da manhã, Arnaldo consegue enviar uma
> seqüência de 24 nuvens. Como Beatriz nem sempre consegue distinguir uma
> nuvem pequena de uma grande, ela e Arnaldo fizeram um dicionário antes de
> ir para o acampamento. No dicionário aparecem N seqüências de 24 tamanhos
> de nuvem (como por exemplo a seqüência PGPGPGPGPGPGGPGPGPGPGPGP, onde G
> significa nuvem grande e P significa nuvem pequena). Para cada uma das N
> seqüências, o dicionário indica seu significado. Para evitar interpretações
> erradas, Arnaldo e Beatriz evitaram incluir no dicionário seqüências
> parecidas. Mais precisamente, duas seqüências no dicionário sempre diferem
> em pelo menos 8 das 24 posições.
>
>
>
> Demonstre que N<= 4096
>
> --
> Esta mensagem foi verificada pelo sistema de antivírus e
> acredita-se estar livre de perigo.

-- 
Esta mensagem foi verificada pelo sistema de antiv�rus e
 acredita-se estar livre de perigo.



[obm-l] OBM 2002 - Problema 6

2019-01-12 Por tôpico Jeferson Almir
Amigos peço ajuda nesse problema, e me orientaram a estudar Códigos
Corretores de Erros.


Arnaldo e Beatriz se comunicam durante um acampamento usando sinais de
fumaça, às vezes usando uma nuvem grande, às vezes uma pequena.

No tempo disponível antes do café da manhã, Arnaldo consegue enviar uma
seqüência de 24 nuvens. Como Beatriz nem sempre consegue distinguir uma
nuvem pequena de uma grande, ela e Arnaldo fizeram um dicionário antes de
ir para o acampamento. No dicionário aparecem N seqüências de 24 tamanhos
de nuvem (como por exemplo a seqüência PGPGPGPGPGPGGPGPGPGPGPGP, onde G
significa nuvem grande e P significa nuvem pequena). Para cada uma das N
seqüências, o dicionário indica seu significado. Para evitar interpretações
erradas, Arnaldo e Beatriz evitaram incluir no dicionário seqüências
parecidas. Mais precisamente, duas seqüências no dicionário sempre diferem
em pelo menos 8 das 24 posições.



Demonstre que N<= 4096

-- 
Esta mensagem foi verificada pelo sistema de antiv�rus e
 acredita-se estar livre de perigo.



[obm-l] Re: [obm-l] [obm-l] Problema olimpíada de maio

2018-05-10 Por tôpico Pedro José
Boa noite!
Múltiplos de 56 tem como últimos algarismos 0, 2, 4, 6 e 8.
Vamos escolher 8 para começar, pois é o que tem a chance de ter o maior
número de algarismos.
Para ter 8 algarismos 12345678, deveria ser múltiplo de 56. Mas 4 não
divide 78 então não pode ser múltiplo de 56(7×8).
Então vamos tentar com 7 algarismos. 1234568
Como 8 | 568. Basta testar para 7.
123456 - 16 = 123440
12344
1234-8=1226
122-12=110
11
7 não divide 11 Então não serve.
Vamos tentar com 6 algarismos.
234568
Basta ver para 7, para 8 Já vimos que 8 | 568.
23456-16=  23440
2344
234-8=226
22-12=10 e 7 não divide 10.
Não presta.
134568.
Basta ver para 7 como já comentado.
13456-16=13440
1344
134-8=126
12-12= 0. Então serve.
O único que pode ter 6 algarismos é um número de final 6.
Porém seria um número  <=123456 <134568
Logo 134568 é o maior múltiplo de 56 que atende.
Deve ter um jeito mais elegante de resolver, usando congruência. Mas está
resolvido.

Saudações,
PJMS

Em 10 de mai de 2018 8:52 PM, "Arthur Vieira" 
escreveu:

> preciso de ajuda com esse problema
>
>
> PROBLEMA 1
>
> Dizemos que um número inteiro positivo é ascendente se seus dígitos
> lidos da esquerda para a direita estão em ordem estritamente crescente.
> Por exemplo, 458 é ascendente e 2339 não é.
> Determine o maior número ascendente que é múltiplo de 56.
>
>
>
> --
> Esta mensagem foi verificada pelo sistema de antivírus e
> acredita-se estar livre de perigo.

-- 
Esta mensagem foi verificada pelo sistema de antiv�rus e
 acredita-se estar livre de perigo.



[obm-l] [obm-l] Problema olimpíada de maio

2018-05-10 Por tôpico Arthur Vieira
preciso de ajuda com esse problema


PROBLEMA 1

Dizemos que um número inteiro positivo é ascendente se seus dígitos
lidos da esquerda para a direita estão em ordem estritamente crescente.
Por exemplo, 458 é ascendente e 2339 não é.
Determine o maior número ascendente que é múltiplo de 56.

-- 
Esta mensagem foi verificada pelo sistema de antiv�rus e
 acredita-se estar livre de perigo.



[obm-l] [obm - l] Re: Limite

2018-03-20 Por tôpico Artur Steiner
> Sendo (a_n) o ln da sequência dada, então 
> 
> a_n = 1/n ln(n!/n^n) = -1/n [-ln(1/n) - ln(2/n)  - ln(n/n)]
> 
> Temos uma sequência de somas inferiores de Riemann sobre [0, 1] da função 
> -ln, correspondente a uma partição de norma 1/n ---> 0. Conforme sabemos da 
> Análise, se a integral imprópria desta funçao sobre [0, 1] convergir, então 
> as somas inferiores convergirão para esta integral. E isto de fato ocorre, 
> pois 
> 
> Int [0, 1] lnx dx = [x lnx - x] [0, 1] = 1 * 0 - 1 - (0 - 0) = -1, visto que 
> lim x ---> 0+ x lnx = 0. Logo, a_n ---> -1e sua sequência converge para 
> e^(-1) = 1/e
> 
> Artur

Enviado do meu iPad
-- 
Esta mensagem foi verificada pelo sistema de antiv�rus e
 acredita-se estar livre de perigo.



[obm-l] OBM-U

2017-08-27 Por tôpico Israel Meireles Chrisostomo
Olá pessoal, eu estou querendo fazer a OBM-U, mas preciso de um professor
para me ajudar a resolver os problemas e me ensinar técnicas e
táticas.Alguém aí estaria interessado em me ajudar?As aulas poderiam ser
pelo skype.
-- 
Israel Meireles Chrisostomo

-- 
Esta mensagem foi verificada pelo sistema de antiv�rus e
 acredita-se estar livre de perigo.



[obm-l] Re: [obm-l] [obm-l] Torneio de Tênis( problema de grafos)

2017-03-24 Por tôpico Mauricio de Araujo
2017-03-23 21:45 GMT-03:00 Pedro Soares :

> Em um torneio de tênis com 14 jogadores


​bom dia,

ver resposta em
http://www.urantiagaia.org/educacional/matematica/combinatoria2/Aula19-MiscelaneaII.pdf
​


--
Abraços,
Mauricio de Araujo
[oɾnɐɹɐ ǝp oıɔıɹnɐɯ]

-- 
Esta mensagem foi verificada pelo sistema de antiv�rus e
 acredita-se estar livre de perigo.



[obm-l] [obm-l] Torneio de Tênis( problema de grafos)

2017-03-23 Por tôpico Pedro Soares
Alguém pode dar alguma ideia?

Em um torneio de tênis com 14 jogadores, cada um joga com todos os outros
exatamente uma vez e não há empates. Prove que é possível escolher 3
jogadores para os quais qualquer um dos outros 11 perdeu para pelo menos um
desses 3.

-- 
Esta mensagem foi verificada pelo sistema de antiv�rus e
 acredita-se estar livre de perigo.



[obm-l] Re: [obm-l] [obm-l] Questão Geometria

2016-10-10 Por tôpico Carlos Victor
 

Olá Vinicius, 

Seja R a intersecção de AO com BC. Seja T a intersecção da bissetriz de
 Será que alguém poria me ajudar na seguinte questão? 
> 
> * 
> 
> (Belarus) Seja O o centro do círculo ex-inscrito do triângulo ABC oposto ao 
> vértice A. Seja M o ponto médio de AC e seja P a intersec ̧ão das retas MO e 
> BC. Prove que se ∠BAC = 2∠ACB, então AB = BP. 
> 
> -- 
> Esta mensagem foi verificada pelo sistema de antivrus e 
> acredita-se estar livre de perigo.
 
-- 
Esta mensagem foi verificada pelo sistema de antiv�rus e
 acredita-se estar livre de perigo.



[obm-l] Re: [obm-l] Re: [obm-l] Re: [obm-l] [obm-l] Questão Geometria

2016-10-09 Por tôpico Jeferson Almir
Sei que o tópico não tem nada a ver com o problema proposto, mas já postei
2 problemas que não aparecem na caixa da lista e percebi que alguns
receberam pois até responderam. Isso já aconteceu com alguém???

Em 9 de outubro de 2016 15:23, Israel Meireles Chrisostomo <
israelmchrisost...@gmail.com> escreveu:

> Se vc não quiser receber mais emails da obm l envie um emeail para obm l
>
> Em 8 de outubro de 2016 13:15, Matheus Herculano <
> matheusherculan...@gmail.com> escreveu:
>
>> A resposta é para de me mandar isso
>>
>> Em 1 de out de 2016 20:00, "vinicius raimundo" 
>> escreveu:
>>
>>> Será que alguém poria me ajudar na seguinte questão?
>>>
>>>
>>>1.
>>>
>>>(Belarus) Seja O o centro do círculo ex-inscrito do triângulo ABC oposto
>>>ao vértice A. Seja M o ponto médio de AC e seja P a intersec ̧ão das
>>>retas MO e BC. Prove que se ∠BAC = 2∠ACB, então  AB = BP.
>>>
>>>
>>> --
>>> Esta mensagem foi verificada pelo sistema de antivírus e
>>> acredita-se estar livre de perigo.
>>
>>
>> --
>> Esta mensagem foi verificada pelo sistema de antivírus e
>> acredita-se estar livre de perigo.
>>
>
>
> --
> Esta mensagem foi verificada pelo sistema de antivírus e
> acredita-se estar livre de perigo.
>

-- 
Esta mensagem foi verificada pelo sistema de antiv�rus e
 acredita-se estar livre de perigo.



[obm-l] Re: [obm-l] Re: [obm-l] Re: [obm-l] [obm-l] Questão Geometria

2016-10-09 Por tôpico Jeferson Almir
Perdão foi processado sim na Mail Archive acabo de constatar mas demorou
alguns dias para aparecer. Valeu!!

Em 9 de outubro de 2016 17:40, Jeferson Almir 
escreveu:

> Sei que o tópico não tem nada a ver com o problema proposto, mas já postei
> 2 problemas que não aparecem na caixa da lista e percebi que alguns
> receberam pois até responderam. Isso já aconteceu com alguém???
>
> Em 9 de outubro de 2016 15:23, Israel Meireles Chrisostomo <
> israelmchrisost...@gmail.com> escreveu:
>
>> Se vc não quiser receber mais emails da obm l envie um emeail para obm l
>>
>> Em 8 de outubro de 2016 13:15, Matheus Herculano <
>> matheusherculan...@gmail.com> escreveu:
>>
>>> A resposta é para de me mandar isso
>>>
>>> Em 1 de out de 2016 20:00, "vinicius raimundo" 
>>> escreveu:
>>>
 Será que alguém poria me ajudar na seguinte questão?


1.

(Belarus) Seja O o centro do círculo ex-inscrito do triângulo ABC oposto
ao vértice A. Seja M o ponto médio de AC e seja P a intersec ̧ão
das retas MO e BC. Prove que se ∠BAC = 2∠ACB, então  AB = BP.


 --
 Esta mensagem foi verificada pelo sistema de antivírus e
 acredita-se estar livre de perigo.
>>>
>>>
>>> --
>>> Esta mensagem foi verificada pelo sistema de antivírus e
>>> acredita-se estar livre de perigo.
>>>
>>
>>
>> --
>> Esta mensagem foi verificada pelo sistema de antivírus e
>> acredita-se estar livre de perigo.
>>
>
>

-- 
Esta mensagem foi verificada pelo sistema de antiv�rus e
 acredita-se estar livre de perigo.



[obm-l] Re: [obm-l] Re: [obm-l] [obm-l] Questão Geometria

2016-10-09 Por tôpico Israel Meireles Chrisostomo
Se vc não quiser receber mais emails da obm l envie um emeail para obm l

Em 8 de outubro de 2016 13:15, Matheus Herculano <
matheusherculan...@gmail.com> escreveu:

> A resposta é para de me mandar isso
>
> Em 1 de out de 2016 20:00, "vinicius raimundo" 
> escreveu:
>
>> Será que alguém poria me ajudar na seguinte questão?
>>
>>
>>1.
>>
>>(Belarus) Seja O o centro do círculo ex-inscrito do triângulo ABC oposto
>>ao vértice A. Seja M o ponto médio de AC e seja P a intersec ̧ão das
>>retas MO e BC. Prove que se ∠BAC = 2∠ACB, então  AB = BP.
>>
>>
>> --
>> Esta mensagem foi verificada pelo sistema de antivírus e
>> acredita-se estar livre de perigo.
>
>
> --
> Esta mensagem foi verificada pelo sistema de antivírus e
> acredita-se estar livre de perigo.
>

-- 
Esta mensagem foi verificada pelo sistema de antiv�rus e
 acredita-se estar livre de perigo.



[obm-l] Re: [obm-l] [obm-l] Questão Geometria

2016-10-08 Por tôpico Matheus Herculano
A resposta é para de me mandar isso

Em 1 de out de 2016 20:00, "vinicius raimundo" 
escreveu:

> Será que alguém poria me ajudar na seguinte questão?
>
>
>1.
>
>(Belarus) Seja O o centro do círculo ex-inscrito do triângulo ABC oposto
>ao vértice A. Seja M o ponto médio de AC e seja P a intersec ̧ão das
>retas MO e BC. Prove que se ∠BAC = 2∠ACB, então  AB = BP.
>
>
> --
> Esta mensagem foi verificada pelo sistema de antivírus e
> acredita-se estar livre de perigo.

-- 
Esta mensagem foi verificada pelo sistema de antiv�rus e
 acredita-se estar livre de perigo.



[obm-l] Re: [obm-l] [obm-l] Questão Geometria

2016-10-04 Por tôpico Douglas Oliveira de Lima
Bom vamos lá, não tem nada de bonito nessa resolução.

Seja O o centro do ex-incirculo de ABC tangente ao lado BC, temos que AO é
bissetriz do ângulo BAC, seja Q a intercessão de AO com BC, e J o pé da
perpendicular tirada de O ao lado AC, sendo BAQ=x, nós teremos CAQ=ACB=x,
AQB=OQC=2x. E OC é bissetriz de BCJ, assim BCO=90-x/2, e sendo P a
intercessao de MO com BC.

1)Aplicando lei dos senos no triângulo AQC teremos

AQ/AC=senx/sen(2x)

2)Agora aplicando no triângulo AMO teremos

AM/MO=sen(QOP)/senx

3)E no triângulo CMO novamente lei dos senos teremos

MC/MO=sen(COP)/cos(x/2)

4)Como AM=MC, dos itens (2) e (3) segue que

sen(QOP)/sen(COP)=senx/cox(x/2)

5) Para o triângulo QPO, nós teremos

sen(QOP)=[(QP)sen(2x)]/PO

6) Para o triângulo CPO, nós teremos

sen(COP)=[(CP).cos(x/2)]/PO

7)Dos itens (5) e (6) podemos concluir que

sen(QOP)/sen(COP)=[(QP).sen(2x)]/[(CP).cos(x/2)]

8)E de (4) e (7) nós temos

senx/cos(x/2)=[(QP).sen(2x)]/[(CP).cos(x/2)], ou melhor QP/CP=senx/sen(2x)

9)Agora de (1) e (8) AQ/AC=QP/CP, donde vem

QAP=CAP e BAP=x+QAP=x+CAP=BPA, ou seja ABP é isosceles e AB=BP.


Um abraço  do
Douglas Oliveira.

Em 1 de outubro de 2016 19:54, vinicius raimundo 
escreveu:

> Será que alguém poria me ajudar na seguinte questão?
>
>
>1.
>
>(Belarus) Seja O o centro do círculo ex-inscrito do triângulo ABC oposto
>ao vértice A. Seja M o ponto médio de AC e seja P a intersec ̧ão das
>retas MO e BC. Prove que se ∠BAC = 2∠ACB, então  AB = BP.
>
>
> --
> Esta mensagem foi verificada pelo sistema de antivírus e
> acredita-se estar livre de perigo.

-- 
Esta mensagem foi verificada pelo sistema de antiv�rus e
 acredita-se estar livre de perigo.



[obm-l] [obm-l] Questão Geometria

2016-10-01 Por tôpico vinicius raimundo
Será que alguém poria me ajudar na seguinte questão?


   1.

   (Belarus) Seja O o centro do círculo ex-inscrito do triângulo ABC oposto
   ao vértice A. Seja M o ponto médio de AC e seja P a intersec ̧ão das
   retas MO e BC. Prove que se ∠BAC = 2∠ACB, então  AB = BP.

-- 
Esta mensagem foi verificada pelo sistema de antiv�rus e
 acredita-se estar livre de perigo.



[obm-l] Re: [obm-l] [obm-l] Polinômios

2016-09-06 Por tôpico saulo nilson
p(1\2)=4
(1\4-1\2)4=R=-1

2016-08-02 18:29 GMT-03:00 Daniel Rocha :

> Alguém poderia, por favor, solucionar o problema abaixo:
>
> O resto da divisão de um polinômio P(x) por (2x - 1) é 4; deste modo, o
> resto da divisão de (x^2 - x)*P(x) por (2x - 1) é:
>
> a) -2
> b) -1/2
> c) 1/2
> d) 2
> e) 4
>
> --
> Esta mensagem foi verificada pelo sistema de antivírus e
> acredita-se estar livre de perigo.

-- 
Esta mensagem foi verificada pelo sistema de antiv�rus e
 acredita-se estar livre de perigo.



[obm-l] Re: [obm-l] Re: [obm-l] Re: [obm-l] [obm-l] Simulado ITA (Equação Modular)

2016-08-22 Por tôpico Ricardo Leão
Muito Obrigado amigos Carlos e Ralph !

Em 22 de agosto de 2016 17:36, Carlos Gomes  escreveu:

> Uma boa alternativa é esboçar as representações gráficas das funções
> f(x)=||x+1|-2 e g(x)=sqrt{x+4} (que são relativamente simples de esboçar) e
> ver que há 4 pontos de interseção; um entre -4 e -3, outros dois entre -2 e
> 0 e mais um entre 3 e 4.
>
> Abraço, Cgomes.
>
> Em 22 de agosto de 2016 16:58, Ralph Teixeira 
> escreveu:
>
>> Confira as suas contas -- cada uma daquelas 4 equacoes tem uma raiz real
>> valida.
>>
>> Abraco, Ralph.
>>
>>
>> 2016-08-22 16:38 GMT-03:00 Ricardo Leão :
>>
>>> Olá amigos,
>>>
>>> Eu gostaria que algum amigo corrigisse a solução que eu desenvolvi para
>>> o seguinte problema envolvendo módulo:
>>>
>>> (Enunciado) O numero de soluções reais da equação | |x+1| - 2 | =
>>> \sqrt{x+4} é:
>>>
>>> a) 0 b) 1 c) 2  d) 3   e) 4
>>>
>>> (MINHA SOLUÇÃO):
>>>
>>> |x+1| = 2 + \sqrt{x+4} ou  |x+1| = 2 - \sqrt{x+4}
>>>
>>> x + 1 = 2 + \sqrt{x+4}   x + 1 = 2 - \sqrt{x+4}
>>> ouou
>>> x + 1 = -2 - \sqrt{x+4}  x + 1 = -2 + \sqrt{x+4}
>>>
>>> Eu chequei e se eu não estiver enganado, o número de soluções é zero.
>>>
>>> Mas de acordo com o gabarito oficial o resultado é 4(item E).
>>>
>>> Eu agradeço muito se alguém me ajudar com essa questão.
>>>
>>> --
>>> Esta mensagem foi verificada pelo sistema de antivírus e
>>> acredita-se estar livre de perigo.
>>
>>
>>
>> --
>> Esta mensagem foi verificada pelo sistema de antivírus e
>> acredita-se estar livre de perigo.
>>
>
>
> --
> Esta mensagem foi verificada pelo sistema de antivírus e
> acredita-se estar livre de perigo.
>

-- 
Esta mensagem foi verificada pelo sistema de antiv�rus e
 acredita-se estar livre de perigo.



[obm-l] Re: [obm-l] Re: [obm-l] [obm-l] Simulado ITA (Equação Modular)

2016-08-22 Por tôpico Carlos Gomes
Uma boa alternativa é esboçar as representações gráficas das funções
f(x)=||x+1|-2 e g(x)=sqrt{x+4} (que são relativamente simples de esboçar) e
ver que há 4 pontos de interseção; um entre -4 e -3, outros dois entre -2 e
0 e mais um entre 3 e 4.

Abraço, Cgomes.

Em 22 de agosto de 2016 16:58, Ralph Teixeira  escreveu:

> Confira as suas contas -- cada uma daquelas 4 equacoes tem uma raiz real
> valida.
>
> Abraco, Ralph.
>
>
> 2016-08-22 16:38 GMT-03:00 Ricardo Leão :
>
>> Olá amigos,
>>
>> Eu gostaria que algum amigo corrigisse a solução que eu desenvolvi para o
>> seguinte problema envolvendo módulo:
>>
>> (Enunciado) O numero de soluções reais da equação | |x+1| - 2 | =
>> \sqrt{x+4} é:
>>
>> a) 0 b) 1 c) 2  d) 3   e) 4
>>
>> (MINHA SOLUÇÃO):
>>
>> |x+1| = 2 + \sqrt{x+4} ou  |x+1| = 2 - \sqrt{x+4}
>>
>> x + 1 = 2 + \sqrt{x+4}   x + 1 = 2 - \sqrt{x+4}
>> ouou
>> x + 1 = -2 - \sqrt{x+4}  x + 1 = -2 + \sqrt{x+4}
>>
>> Eu chequei e se eu não estiver enganado, o número de soluções é zero.
>>
>> Mas de acordo com o gabarito oficial o resultado é 4(item E).
>>
>> Eu agradeço muito se alguém me ajudar com essa questão.
>>
>> --
>> Esta mensagem foi verificada pelo sistema de antivírus e
>> acredita-se estar livre de perigo.
>
>
>
> --
> Esta mensagem foi verificada pelo sistema de antivírus e
> acredita-se estar livre de perigo.
>

-- 
Esta mensagem foi verificada pelo sistema de antiv�rus e
 acredita-se estar livre de perigo.



[obm-l] Re: [obm-l] [obm-l] Simulado ITA (Equação Modular)

2016-08-22 Por tôpico Ralph Teixeira
Confira as suas contas -- cada uma daquelas 4 equacoes tem uma raiz real
valida.

Abraco, Ralph.


2016-08-22 16:38 GMT-03:00 Ricardo Leão :

> Olá amigos,
>
> Eu gostaria que algum amigo corrigisse a solução que eu desenvolvi para o
> seguinte problema envolvendo módulo:
>
> (Enunciado) O numero de soluções reais da equação | |x+1| - 2 | =
> \sqrt{x+4} é:
>
> a) 0 b) 1 c) 2  d) 3   e) 4
>
> (MINHA SOLUÇÃO):
>
> |x+1| = 2 + \sqrt{x+4} ou  |x+1| = 2 - \sqrt{x+4}
>
> x + 1 = 2 + \sqrt{x+4}   x + 1 = 2 - \sqrt{x+4}
> ouou
> x + 1 = -2 - \sqrt{x+4}  x + 1 = -2 + \sqrt{x+4}
>
> Eu chequei e se eu não estiver enganado, o número de soluções é zero.
>
> Mas de acordo com o gabarito oficial o resultado é 4(item E).
>
> Eu agradeço muito se alguém me ajudar com essa questão.
>
> --
> Esta mensagem foi verificada pelo sistema de antivírus e
> acredita-se estar livre de perigo.

-- 
Esta mensagem foi verificada pelo sistema de antiv�rus e
 acredita-se estar livre de perigo.



[obm-l] [obm-l] Simulado ITA (Equação Modular)

2016-08-22 Por tôpico Ricardo Leão
Olá amigos,

Eu gostaria que algum amigo corrigisse a solução que eu desenvolvi para o
seguinte problema envolvendo módulo:

(Enunciado) O numero de soluções reais da equação | |x+1| - 2 | =
\sqrt{x+4} é:

a) 0 b) 1 c) 2  d) 3   e) 4

(MINHA SOLUÇÃO):

|x+1| = 2 + \sqrt{x+4} ou  |x+1| = 2 - \sqrt{x+4}

x + 1 = 2 + \sqrt{x+4}   x + 1 = 2 - \sqrt{x+4}
ouou
x + 1 = -2 - \sqrt{x+4}  x + 1 = -2 + \sqrt{x+4}

Eu chequei e se eu não estiver enganado, o número de soluções é zero.

Mas de acordo com o gabarito oficial o resultado é 4(item E).

Eu agradeço muito se alguém me ajudar com essa questão.

-- 
Esta mensagem foi verificada pelo sistema de antiv�rus e
 acredita-se estar livre de perigo.



[obm-l] Re: [obm-l] Re: [obm-l] Re: [obm-l] [obm-l] Domínio de Função Modular

2016-08-19 Por tôpico Pedro José
Boa tarde!

Se o problema for como aprsentado, deve-se procurar o domínio mais amplo em
|R, o que daria |R - {-3,3}.
Agora, se houver restrição que a função é definida para  D em |R-, aí a
resposta do gabarito está correta.
Reveja o enunciado na íntegra e veja se não há nada que force a imagem a
ser um subconjunto de |R-.
Pois a restrição apontada é para que f(x) < 0 .

Saudações,
PJMS

Em 19 de agosto de 2016 13:50, Ricardo Leão 
escreveu:

> Olá Israel,
>
> A função é essa mesma:
>
> f(x) = 1 / (|x| - 3) (fração)
>
> Eu gostaria de saber se sou eu ou se é o gabarito que está errado.
>
> Em 19 de agosto de 2016 12:51, Israel Meireles Chrisostomo <
> israelmchrisost...@gmail.com> escreveu:
>
>> Olha eu acho que deve ter alguma coisa errada com o que vc escreveu, por
>> acaso |x|-3  está dentro de uma raíz?talvez vc tenha se esquecido de
>> colocar...
>>
>> Em 19 de agosto de 2016 12:19, Ricardo Leão 
>> escreveu:
>>
>>> Olá amigos,
>>>
>>> Eu me deparei com uma questão acerca de domínio de função modular.
>>> Ela pede o domínio da função f(x) = 1 / (|x| - 3).
>>> Que eu saiba o domínio deveria ser { x pertencente aos reais | x
>>> diferente de -3 ou x diferente de 3}.
>>> No entanto, de acordo com o gabarito oficial o domínio é -3 < x < 3.
>>>
>>> Alguém poderia, por favor, me explicar por quê ?
>>>
>>> --
>>> Esta mensagem foi verificada pelo sistema de antivírus e
>>> acredita-se estar livre de perigo.
>>
>>
>>
>> --
>> Esta mensagem foi verificada pelo sistema de antivírus e
>> acredita-se estar livre de perigo.
>
>
>
> --
> Esta mensagem foi verificada pelo sistema de antivírus e
> acredita-se estar livre de perigo.
>

-- 
Esta mensagem foi verificada pelo sistema de antiv�rus e
 acredita-se estar livre de perigo.



[obm-l] Re: [obm-l] Re: [obm-l] [obm-l] Domínio de Função Modular

2016-08-19 Por tôpico Ricardo Leão
Olá Israel,

A função é essa mesma:

f(x) = 1 / (|x| - 3) (fração)

Eu gostaria de saber se sou eu ou se é o gabarito que está errado.

Em 19 de agosto de 2016 12:51, Israel Meireles Chrisostomo <
israelmchrisost...@gmail.com> escreveu:

> Olha eu acho que deve ter alguma coisa errada com o que vc escreveu, por
> acaso |x|-3  está dentro de uma raíz?talvez vc tenha se esquecido de
> colocar...
>
> Em 19 de agosto de 2016 12:19, Ricardo Leão 
> escreveu:
>
>> Olá amigos,
>>
>> Eu me deparei com uma questão acerca de domínio de função modular.
>> Ela pede o domínio da função f(x) = 1 / (|x| - 3).
>> Que eu saiba o domínio deveria ser { x pertencente aos reais | x
>> diferente de -3 ou x diferente de 3}.
>> No entanto, de acordo com o gabarito oficial o domínio é -3 < x < 3.
>>
>> Alguém poderia, por favor, me explicar por quê ?
>>
>> --
>> Esta mensagem foi verificada pelo sistema de antivírus e
>> acredita-se estar livre de perigo.
>
>
>
> --
> Esta mensagem foi verificada pelo sistema de antivírus e
> acredita-se estar livre de perigo.

-- 
Esta mensagem foi verificada pelo sistema de antiv�rus e
 acredita-se estar livre de perigo.



[obm-l] Re: [obm-l] [obm-l] Domínio de Função Modular

2016-08-19 Por tôpico Israel Meireles Chrisostomo
Olha eu acho que deve ter alguma coisa errada com o que vc escreveu, por
acaso |x|-3  está dentro de uma raíz?talvez vc tenha se esquecido de
colocar...

Em 19 de agosto de 2016 12:19, Ricardo Leão 
escreveu:

> Olá amigos,
>
> Eu me deparei com uma questão acerca de domínio de função modular.
> Ela pede o domínio da função f(x) = 1 / (|x| - 3).
> Que eu saiba o domínio deveria ser { x pertencente aos reais | x diferente
> de -3 ou x diferente de 3}.
> No entanto, de acordo com o gabarito oficial o domínio é -3 < x < 3.
>
> Alguém poderia, por favor, me explicar por quê ?
>
> --
> Esta mensagem foi verificada pelo sistema de antivírus e
> acredita-se estar livre de perigo.

-- 
Esta mensagem foi verificada pelo sistema de antiv�rus e
 acredita-se estar livre de perigo.



[obm-l] [obm-l] Domínio de Função Modular

2016-08-19 Por tôpico Ricardo Leão
Olá amigos,

Eu me deparei com uma questão acerca de domínio de função modular.
Ela pede o domínio da função f(x) = 1 / (|x| - 3).
Que eu saiba o domínio deveria ser { x pertencente aos reais | x diferente
de -3 ou x diferente de 3}.
No entanto, de acordo com o gabarito oficial o domínio é -3 < x < 3.

Alguém poderia, por favor, me explicar por quê ?

-- 
Esta mensagem foi verificada pelo sistema de antiv�rus e
 acredita-se estar livre de perigo.



[obm-l] Re: [obm-l] Re: [obm-l] [obm-l] Polinômios

2016-08-08 Por tôpico Pedro José
Desculpe-me,

4x^2 - 4x = (2x - 1) (2x-1) -1. O resto é: -1 opção inexistente.

Se usar P(x) = q(x) * (2x-1) + 4  e aplicar em x = - 1/2.

P(-1/2) = 4.

P(x). (x^2-2x) = q1(x) * (2x-1) + r, novamente aplicando em -1/2.
P(-1/2) * (-1/4) = r
4* - 1/4 = r ==> r = -1

Não há opção, ou o enunciado ou a lista de resposta está incorreto, ou o
problema não está bem formulado.



Em 8 de agosto de 2016 09:22, Pedro José  escreveu:

> Bom dia!
>
> (i) P(x) = q(x) * (2x-1) + 4 onde q(x) é um polinômio, porque o resto da
> divisão de P(x) por (2x-1) é 4, pelo enunciado.
>
> Multiplicando por (x^2-x) dos dois lados da igualde (i), temos;
>
> (x^2-x) * P(x) =  (x^2-x) *  [q(x) * (2x-1) + 4]
> (x^2-x) * P(x) = (x^2-x) * q(x) * (2x - 1) + 4 * (x^2-x)
> (x^2-x) * P(x) = (x^2-x) * q(x) * (2x - 1) + 4x^2 - 4X (ii)
>
> 4x^2 -4x = (2x-2) (2x-1) -2 (iii)
>
> (iii) aplicado em (ii) ==> (x^2-x) * P(x) = (x^2-x) * q(x) * (2x - 1) +
> (2x-2) (2x-1) -2
>
> (x^2-x) * P(x) = [(x^2-x) * q(x) + (2x-2)] * (2x-1) -2 (iv)
>
> Pelo fechamento da multiplicação e adição de polinômios, (x^2-x) * q(x) +
> (2x-2) é um polinômio, seja q1(x) = (x^2-x) * q(x) + (2x-2) (v)
>
> (v) em (iv) ==>  (x^2-x) * P(x) = q1(x) * (2x - 1) - 2 , logo o resto é
> -2. Opção *a)*
>
>
>
> Em 4 de agosto de 2016 00:17, Tarsis Esau  escreveu:
>
>> Oi. Ótimas dicas, mas minha resposta não bate com nenhuma das
>> alternativas.
>> --
>> Esta mensagem foi verificada pelo sistema de antivírus e
>> acredita-se estar livre de perigo.
>>
>
>

-- 
Esta mensagem foi verificada pelo sistema de antiv�rus e
 acredita-se estar livre de perigo.



[obm-l] Re: [obm-l] Re: [obm-l] [obm-l] Polinômios

2016-08-08 Por tôpico Pedro José
Bom dia!

(i) P(x) = q(x) * (2x-1) + 4 onde q(x) é um polinômio, porque o resto da
divisão de P(x) por (2x-1) é 4, pelo enunciado.

Multiplicando por (x^2-x) dos dois lados da igualde (i), temos;

(x^2-x) * P(x) =  (x^2-x) *  [q(x) * (2x-1) + 4]
(x^2-x) * P(x) = (x^2-x) * q(x) * (2x - 1) + 4 * (x^2-x)
(x^2-x) * P(x) = (x^2-x) * q(x) * (2x - 1) + 4x^2 - 4X (ii)

4x^2 -4x = (2x-2) (2x-1) -2 (iii)

(iii) aplicado em (ii) ==> (x^2-x) * P(x) = (x^2-x) * q(x) * (2x - 1) +
(2x-2) (2x-1) -2

(x^2-x) * P(x) = [(x^2-x) * q(x) + (2x-2)] * (2x-1) -2 (iv)

Pelo fechamento da multiplicação e adição de polinômios, (x^2-x) * q(x) +
(2x-2) é um polinômio, seja q1(x) = (x^2-x) * q(x) + (2x-2) (v)

(v) em (iv) ==>  (x^2-x) * P(x) = q1(x) * (2x - 1) - 2 , logo o resto é -2.
Opção *a)*



Em 4 de agosto de 2016 00:17, Tarsis Esau  escreveu:

> Oi. Ótimas dicas, mas minha resposta não bate com nenhuma das
> alternativas.
> --
> Esta mensagem foi verificada pelo sistema de antivírus e
> acredita-se estar livre de perigo.
>

-- 
Esta mensagem foi verificada pelo sistema de antiv�rus e
 acredita-se estar livre de perigo.



[obm-l] Re: [obm-l] [obm-l] Polinômios

2016-08-03 Por tôpico Tarsis Esau
Oi. Ótimas dicas, mas minha resposta não bate com nenhuma das alternativas.

-- 
Esta mensagem foi verificada pelo sistema de antiv�rus e
 acredita-se estar livre de perigo.



[obm-l] Re: [obm-l] Re: [obm-l] Re: [obm-l] [obm-l] Polinômios

2016-08-03 Por tôpico Pedro José
Boa tarde!

Só ter a resposta, você irá apresentá-la para o professor. Mas e o próximo.
Tem que ter algum esforço seu para chegar na resposta.

Vamos usar números para facilitar.

O resto de um número k por 9 é 3. Qual o resto de 7k por 3.

Se o resto de k por 9 é 3, exista q inteiro tal que k = 9q + 3 (i)

Se multiplicar os dois lados por 7 terei 7k = 63q + 21
o resto de 21 por 9 é 3 então tenho 21 = 2*9 +3

7k = 63q + 2*9 + 3 ou 7k = 9*(7q+2) + 3

Pelo fechamento da multiplicação e adição em Z 7q + 2 também é inteiro
então o resto é 3.

Podia já ter largado de mão o 63 q pois é múltiplo de 9 e só ver o resto de
21 por 9.

Para os polinômios vale o mesmo princípio

Se o resto de D(x) por d(x) é r(x), então existe um polinômio q(x) de grau
igual ao diferença dos graus de D(x) e d(x) tal que:

D(x) = q(x) . d(x) + r(x).

A soma de polinômios é fechada. Pois, dois polinômios somados dão um
polinômio.
O produto de polinômios é fechado. Pois, o produto de dois polinômios dá um
polinômio.
Dá uma revisada na matéria apresentada pelo professor e tenta resolver o
problema. Você conseguirá.








Em 2 de agosto de 2016 20:15, Daniel Rocha 
escreveu:

> Cara eu não entendi nenhuma das duas explicações.
>
> Qual é o item correto então???
>
> Em 2 de agosto de 2016 19:26, Pedro José  escreveu:
>
>> Boa noite!
>>
>> O resto da divisão de um polinômio P(x) por (2x - 1) é 4 ==> P(x) = q(x)
>> *(2x-1) + 4 (i), onde q(x) é um polinômio com grau igual a grau de P(x) - 1.
>>
>> (x^2- x) * P(x) = (x^2-x) * [q(x) *(2x-1) + 4] - por (i), basta
>> multiplicar ambos os lados da igualdade por (x^2-x) aí você vai ter
>> (x^2- x) * P(x) = q1(x) * (2x-1) + p1(x)
>>
>> Pelo fechamento da multiplicação tanto q1(x), quanto p1(x) são
>> polinômios. então bastará achar o resto de p1(x) por (2x-1)
>>
>> Tente fazer outros exemplos para fixar. .
>>
>>
>> Saudações,
>> PJMS
>>
>>
>>
>>
>> Em 2 de agosto de 2016 18:29, Daniel Rocha 
>> escreveu:
>>
>>> Alguém poderia, por favor, solucionar o problema abaixo:
>>>
>>> O resto da divisão de um polinômio P(x) por (2x - 1) é 4; deste modo, o
>>> resto da divisão de (x^2 - x)*P(x) por (2x - 1) é:
>>>
>>> a) -2
>>> b) -1/2
>>> c) 1/2
>>> d) 2
>>> e) 4
>>>
>>> --
>>> Esta mensagem foi verificada pelo sistema de antivírus e
>>> acredita-se estar livre de perigo.
>>
>>
>>
>> --
>> Esta mensagem foi verificada pelo sistema de antivírus e
>> acredita-se estar livre de perigo.
>>
>
>
> --
> Esta mensagem foi verificada pelo sistema de antivírus e
> acredita-se estar livre de perigo.
>

-- 
Esta mensagem foi verificada pelo sistema de antiv�rus e
 acredita-se estar livre de perigo.



[obm-l] Re: [obm-l] Re: [obm-l] [obm-l] Polinômios

2016-08-02 Por tôpico Daniel Rocha
Cara eu não entendi nenhuma das duas explicações.

Qual é o item correto então???

Em 2 de agosto de 2016 19:26, Pedro José  escreveu:

> Boa noite!
>
> O resto da divisão de um polinômio P(x) por (2x - 1) é 4 ==> P(x) = q(x)
> *(2x-1) + 4 (i), onde q(x) é um polinômio com grau igual a grau de P(x) - 1.
>
> (x^2- x) * P(x) = (x^2-x) * [q(x) *(2x-1) + 4] - por (i), basta
> multiplicar ambos os lados da igualdade por (x^2-x) aí você vai ter
> (x^2- x) * P(x) = q1(x) * (2x-1) + p1(x)
>
> Pelo fechamento da multiplicação tanto q1(x), quanto p1(x) são
> polinômios. então bastará achar o resto de p1(x) por (2x-1)
>
> Tente fazer outros exemplos para fixar. .
>
>
> Saudações,
> PJMS
>
>
>
>
> Em 2 de agosto de 2016 18:29, Daniel Rocha 
> escreveu:
>
>> Alguém poderia, por favor, solucionar o problema abaixo:
>>
>> O resto da divisão de um polinômio P(x) por (2x - 1) é 4; deste modo, o
>> resto da divisão de (x^2 - x)*P(x) por (2x - 1) é:
>>
>> a) -2
>> b) -1/2
>> c) 1/2
>> d) 2
>> e) 4
>>
>> --
>> Esta mensagem foi verificada pelo sistema de antivírus e
>> acredita-se estar livre de perigo.
>
>
>
> --
> Esta mensagem foi verificada pelo sistema de antivírus e
> acredita-se estar livre de perigo.
>

-- 
Esta mensagem foi verificada pelo sistema de antiv�rus e
 acredita-se estar livre de perigo.



[obm-l] Re: [obm-l] [obm-l] Polinômios

2016-08-02 Por tôpico Pedro José
Boa noite!

O resto da divisão de um polinômio P(x) por (2x - 1) é 4 ==> P(x) = q(x)
*(2x-1) + 4 (i), onde q(x) é um polinômio com grau igual a grau de P(x) - 1.

(x^2- x) * P(x) = (x^2-x) * [q(x) *(2x-1) + 4] - por (i), basta multiplicar
ambos os lados da igualdade por (x^2-x) aí você vai ter
(x^2- x) * P(x) = q1(x) * (2x-1) + p1(x)

Pelo fechamento da multiplicação tanto q1(x), quanto p1(x) são
polinômios. então bastará achar o resto de p1(x) por (2x-1)

Tente fazer outros exemplos para fixar. .


Saudações,
PJMS




Em 2 de agosto de 2016 18:29, Daniel Rocha 
escreveu:

> Alguém poderia, por favor, solucionar o problema abaixo:
>
> O resto da divisão de um polinômio P(x) por (2x - 1) é 4; deste modo, o
> resto da divisão de (x^2 - x)*P(x) por (2x - 1) é:
>
> a) -2
> b) -1/2
> c) 1/2
> d) 2
> e) 4
>
> --
> Esta mensagem foi verificada pelo sistema de antivírus e
> acredita-se estar livre de perigo.

-- 
Esta mensagem foi verificada pelo sistema de antiv�rus e
 acredita-se estar livre de perigo.



[obm-l] Re: [obm-l] [obm-l] Polinômios

2016-08-02 Por tôpico Bernardo Freitas Paulo da Costa
2016-08-02 18:29 GMT-03:00 Daniel Rocha :
> Alguém poderia, por favor, solucionar o problema abaixo:
>
> O resto da divisão de um polinômio P(x) por (2x - 1) é 4; deste modo, o
> resto da divisão de (x^2 - x)*P(x) por (2x - 1) é:

Relacione o resto da divisão com o valor do polinômio em algum "x" "esperto".

Abraços,
-- 
Bernardo Freitas Paulo da Costa

-- 
Esta mensagem foi verificada pelo sistema de antiv�rus e
 acredita-se estar livre de perigo.


=
Instru��es para entrar na lista, sair da lista e usar a lista em
http://www.mat.puc-rio.br/~obmlistas/obm-l.html
=


[obm-l] [obm-l] Polinômios

2016-08-02 Por tôpico Daniel Rocha
Alguém poderia, por favor, solucionar o problema abaixo:

O resto da divisão de um polinômio P(x) por (2x - 1) é 4; deste modo, o
resto da divisão de (x^2 - x)*P(x) por (2x - 1) é:

a) -2
b) -1/2
c) 1/2
d) 2
e) 4

-- 
Esta mensagem foi verificada pelo sistema de antiv�rus e
 acredita-se estar livre de perigo.



[obm-l] Re: [obm-l] Re: [obm-l] [obm-l] Binômio de Newton

2016-08-01 Por tôpico Daniel Rocha
Muito Obrigado pela ajuda, Vinícius!!!

Em 1 de agosto de 2016 20:28, vinicius raimundo 
escreveu:

> Eu entendi o problema desta forma:
>
> O quinto termo da sequência seria
> \binom{n+1}{4}=126, então temos:
>
> (n+1).(n).(n-1).(n-2)=126.4!=3024
>
> Fatorando 3024 vemos que é igual a
> 2^4 . 3^3 . 7
> E como 3024 é o produto de quatro números consecutivos temos:
>
> 9.8.7.6=3024
>
> Logo n=8
>
> end
>
> Em segunda-feira, 1 de agosto de 2016, Daniel Rocha <
> daniel.rocha@gmail.com> escreveu:
>
>> Alguém poderia, por favor, solucionar o problema abaixo:
>>
>> Se o quinto termo da sequência
>> \binom{n+1}{0},\binom{n+1}{1},\binom{n+1}{2},...,\binom{n+1}{n+1} é igual a
>> 126, então o número n é:
>>
>>
>>
>> --
>> Esta mensagem foi verificada pelo sistema de antivírus e
>> acredita-se estar livre de perigo.
>
>
> --
> Esta mensagem foi verificada pelo sistema de antivírus e
> acredita-se estar livre de perigo.

-- 
Esta mensagem foi verificada pelo sistema de antiv�rus e
 acredita-se estar livre de perigo.



[obm-l] Re: [obm-l] [obm-l] Binômio de Newton

2016-08-01 Por tôpico vinicius raimundo
Eu entendi o problema desta forma:

O quinto termo da sequência seria
\binom{n+1}{4}=126, então temos:

(n+1).(n).(n-1).(n-2)=126.4!=3024

Fatorando 3024 vemos que é igual a
2^4 . 3^3 . 7
E como 3024 é o produto de quatro números consecutivos temos:

9.8.7.6=3024

Logo n=8

end

Em segunda-feira, 1 de agosto de 2016, Daniel Rocha <
daniel.rocha@gmail.com> escreveu:

> Alguém poderia, por favor, solucionar o problema abaixo:
>
> Se o quinto termo da sequência
> \binom{n+1}{0},\binom{n+1}{1},\binom{n+1}{2},...,\binom{n+1}{n+1} é igual a
> 126, então o número n é:
>
>
>
> --
> Esta mensagem foi verificada pelo sistema de antivírus e
> acredita-se estar livre de perigo.

-- 
Esta mensagem foi verificada pelo sistema de antiv�rus e
 acredita-se estar livre de perigo.



[obm-l] [obm-l] Binômio de Newton

2016-08-01 Por tôpico Daniel Rocha
Alguém poderia, por favor, solucionar o problema abaixo:

Se o quinto termo da sequência
\binom{n+1}{0},\binom{n+1}{1},\binom{n+1}{2},...,\binom{n+1}{n+1} é igual a
126, então o número n é:

-- 
Esta mensagem foi verificada pelo sistema de antiv�rus e
 acredita-se estar livre de perigo.



Re: [obm-l] [obm-l] Números Complexos

2016-07-11 Por tôpico mathhawk2003
u=wi=>u=(zi)i=>u=zi^2.:u=-z.
(alternativa "a")

 Mensagem original De : Daniel Rocha 
<daniel.rocha@gmail.com> Data:10/07/2016  13:04  (GMT-03:00) 
Para: obm-l@mat.puc-rio.br Assunto: [obm-l] [obm-l] 
Números Complexos 
Alguém poderia, por favor, solucionar o problema abaixo:

Os números complexos z, w, u são tais que w/z = u/w = i (i é a unidade 
imaginária). É correto afirmar que:

a) z é oposto de u.
b) z é o conjugado de u.
c) z é o quadrado de u.
d) z é igual a u.
e) z é igual a u + w.


-- 
Esta mensagem foi verificada pelo sistema de antiv�rus e 
acredita-se estar livre de perigo.
-- 
Esta mensagem foi verificada pelo sistema de antiv�rus e
 acredita-se estar livre de perigo.



[obm-l] Re: [obm-l] Re: [obm-l] [obm-l] Números Complexos

2016-07-10 Por tôpico Daniel Rocha
Muito Obrigado, Carlos !!!

Em 10 de julho de 2016 22:05, Carlos Gomes  escreveu:

> Olá Daniel,
>
> vc faz assim,
>
> Ora, como w/z=u/w=i, segue que w=i.z e u=i.w. Assim,
>
> u=i.w=i.(i.z)=i^2.z=-1.z=-z ==> z=-u , ou seja, z é o posto de u.
> (Alternativa "a")
>
> Abraco, Cgomes.
>
> Em 10 de julho de 2016 13:04, Daniel Rocha 
> escreveu:
>
>> Alguém poderia, por favor, solucionar o problema abaixo:
>>
>> Os números complexos z, w, u são tais que w/z = u/w = i (i é a unidade
>> imaginária). É correto afirmar que:
>>
>> a) z é oposto de u.
>> b) z é o conjugado de u.
>> c) z é o quadrado de u.
>> d) z é igual a u.
>> e) z é igual a u + w.
>>
>>
>> --
>> Esta mensagem foi verificada pelo sistema de antivírus e
>> acredita-se estar livre de perigo.
>
>
>
> --
> Esta mensagem foi verificada pelo sistema de antivírus e
> acredita-se estar livre de perigo.

-- 
Esta mensagem foi verificada pelo sistema de antiv�rus e
 acredita-se estar livre de perigo.



[obm-l] Re: [obm-l] [obm-l] Números Complexos

2016-07-10 Por tôpico Carlos Gomes
Olá Daniel,

vc faz assim,

Ora, como w/z=u/w=i, segue que w=i.z e u=i.w. Assim,

u=i.w=i.(i.z)=i^2.z=-1.z=-z ==> z=-u , ou seja, z é o posto de u.
(Alternativa "a")

Abraco, Cgomes.

Em 10 de julho de 2016 13:04, Daniel Rocha 
escreveu:

> Alguém poderia, por favor, solucionar o problema abaixo:
>
> Os números complexos z, w, u são tais que w/z = u/w = i (i é a unidade
> imaginária). É correto afirmar que:
>
> a) z é oposto de u.
> b) z é o conjugado de u.
> c) z é o quadrado de u.
> d) z é igual a u.
> e) z é igual a u + w.
>
>
> --
> Esta mensagem foi verificada pelo sistema de antivírus e
> acredita-se estar livre de perigo.

-- 
Esta mensagem foi verificada pelo sistema de antiv�rus e
 acredita-se estar livre de perigo.



[obm-l] [obm-l] Números Complexos

2016-07-10 Por tôpico Daniel Rocha
Alguém poderia, por favor, solucionar o problema abaixo:

Os números complexos z, w, u são tais que w/z = u/w = i (i é a unidade
imaginária). É correto afirmar que:

a) z é oposto de u.
b) z é o conjugado de u.
c) z é o quadrado de u.
d) z é igual a u.
e) z é igual a u + w.

-- 
Esta mensagem foi verificada pelo sistema de antiv�rus e
 acredita-se estar livre de perigo.



[obm-l] [obm-l] Geometria Analítica em 3 dimensões

2015-10-30 Por tôpico Rígille Scherrer Borges Menezes
Vc quer dizer de segmento de reta talveZ? Acho que uma boa ideia é usar a
desigualdade triangular.

Em sexta-feira, 30 de outubro de 2015, Israel Meireles Chrisostomo <
israelmchrisost...@gmail.com
> escreveu:

> Olá pessoal alguém sabe como provar que a equação da reta é
> (x_1-x_0)²+(y_1-y_0)²+(z_1-z_0)²=r²? onde r é o comprimento da reta
>
> --
> Esta mensagem foi verificada pelo sistema de antivírus e
> acredita-se estar livre de perigo.

-- 
Esta mensagem foi verificada pelo sistema de antiv�rus e
 acredita-se estar livre de perigo.



[obm-l] Re: [obm-l] [obm-l] Geometria Analítica em 3 dimensões

2015-10-30 Por tôpico Sávio Ribas
Acho que esse livro pode te ajudar:
https://www.dropbox.com/s/jj3xq0hjv2z39zp/gaalt0.pdf

Em 30 de outubro de 2015 15:13, Rígille Scherrer Borges Menezes <
rigillesbmene...@gmail.com> escreveu:

> Vc quer dizer de segmento de reta talveZ? Acho que uma boa ideia é usar a
> desigualdade triangular.
>
> Em sexta-feira, 30 de outubro de 2015, Israel Meireles Chrisostomo <
> israelmchrisost...@gmail.com> escreveu:
>
>> Olá pessoal alguém sabe como provar que a equação da reta é
>> (x_1-x_0)²+(y_1-y_0)²+(z_1-z_0)²=r²? onde r é o comprimento da reta
>>
>> --
>> Esta mensagem foi verificada pelo sistema de antivírus e
>> acredita-se estar livre de perigo.
>
>
> --
> Esta mensagem foi verificada pelo sistema de antivírus e
> acredita-se estar livre de perigo.
>

-- 
Esta mensagem foi verificada pelo sistema de antiv�rus e
 acredita-se estar livre de perigo.



[obm-l] Re: [obm-l] Re: [obm-l] OBM NÍVEL 3 TERCEIRA FASE PRIMEIRO DIA

2014-11-17 Por tôpico Carlos Victor
Oi grande Douglas !!

Como sempre postando bons problemas para a nossa comunidade.

Vamos lá :

Sejam M,N,R e Q os incentros dos triângulos ABP, BPC, CPD e APD
respectivamente.

Sejam S, T, U e V  os incentros dos triângulos ABC,BCD, ACD e ABD
respectivamente.

1º) mostre que  MNRQ é um losango. Mostre também que os raios dos círculos
inscritos em ABC e ADC são iguais; da mesma forma dos triângulos ABD e BCD.

2º) depois mostre que  AM/MS =  AQ/QU  e que  SN/NC = RU/RT .

3º) como consequência SU é paralelo a BD e que VT é paralelo  a AC.

4º) mostre que mostre que MS/AM = UR/RC.

5º) mostre que o ângulo MSN = ângulo QUR ; da mesma forma  ângulo NTR  =
ângulo MVQ.

6º) conclua então que ASCU é um paralelogramo.

7º) conclua daí que pelo fato de PN = PQ e  MP = PR , teremos S pertencente
a BD e  V pertencente a AC.

8º) Como BP é bissetriz e intersecta AC no ponto médio, temos que AB=AC e
BP é perpendicular a BC.

9º) da mesma forma ACD é isósceles.

10º) ídem para BCD e ABD .


Conclusão : ABCD é um losango.. UFA .


Abraços

Carlos  Victor


Em 30 de outubro de 2014 12:22, Esdras Muniz esdrasmunizm...@gmail.com
escreveu:

 Opa, eu tinha entendido círculos circunscritos... Foi mal.

 Em 30 de outubro de 2014 11:02, Esdras Muniz esdrasmunizm...@gmail.com
 escreveu:



 Em 29 de outubro de 2014 22:50, Douglas Oliveira de Lima 
 profdouglaso.del...@gmail.com escreveu:


  *PROBLEMA 1 *

 Seja *ABCD *um quadrilátero convexo e seja *P *a interseção das
 diagonais *AC *e *BD*. Os raios dos círculos inscritos nos triângulos
 *ABP*, *BCP*, *CDP *e *DAP *são iguais. Prove que *ABCD *é um losango.


 Como poderíamos fazer esse problema?




 --
 Esta mensagem foi verificada pelo sistema de antivírus e
 acredita-se estar livre de perigo.




 --
 Esdras Muniz Mota
 Graduando em Matemática Bacharelado
 Universidade Federal do Ceará





 --
 Esdras Muniz Mota
 Graduando em Matemática Bacharelado
 Universidade Federal do Ceará



 --
 Esta mensagem foi verificada pelo sistema de antivírus e
 acredita-se estar livre de perigo.


-- 
Esta mensagem foi verificada pelo sistema de antiv�rus e
 acredita-se estar livre de perigo.



[obm-l] Re: [obm-l] OBM NÍVEL 3 TERCEIRA FASE PRIMEIRO DIA

2014-10-30 Por tôpico Esdras Muniz
Opa, eu tinha entendido círculos circunscritos... Foi mal.

Em 30 de outubro de 2014 11:02, Esdras Muniz esdrasmunizm...@gmail.com
escreveu:



 Em 29 de outubro de 2014 22:50, Douglas Oliveira de Lima 
 profdouglaso.del...@gmail.com escreveu:


  *PROBLEMA 1 *

 Seja *ABCD *um quadrilátero convexo e seja *P *a interseção das
 diagonais *AC *e *BD*. Os raios dos círculos inscritos nos triângulos
 *ABP*, *BCP*, *CDP *e *DAP *são iguais. Prove que *ABCD *é um losango.


 Como poderíamos fazer esse problema?




 --
 Esta mensagem foi verificada pelo sistema de antivírus e
 acredita-se estar livre de perigo.




 --
 Esdras Muniz Mota
 Graduando em Matemática Bacharelado
 Universidade Federal do Ceará





-- 
Esdras Muniz Mota
Graduando em Matemática Bacharelado
Universidade Federal do Ceará

-- 
Esta mensagem foi verificada pelo sistema de antiv�rus e
 acredita-se estar livre de perigo.



[obm-l] OBM NÍVEL 3 TERCEIRA FASE PRIMEIRO DIA

2014-10-29 Por tôpico Douglas Oliveira de Lima
 *PROBLEMA 1 *

Seja *ABCD *um quadrilátero convexo e seja *P *a interseção das diagonais *AC
*e *BD*. Os raios dos círculos inscritos nos triângulos *ABP*, *BCP*, *CDP *e
*DAP *são iguais. Prove que *ABCD *é um losango.


Como poderíamos fazer esse problema?

-- 
Esta mensagem foi verificada pelo sistema de antiv�rus e
 acredita-se estar livre de perigo.



[obm-l] [obm-l] Recorreção - IME

2014-02-18 Por tôpico Sergio Lima
Caro João,


Desculpe retomar esse tema pegando carona numa mensagem sua.


O item (b) da questão 7 da prova discursiva de Matemática do

vestibular do IME de 2013/2014 pede o valor MÍNIMO

da área do triângulo. Pelo menos é isto que está escrito

na versão da prova que baixei do site oficial do IME.

Isto não faz muito sentido. Houve alguma correção no decorrer

da prova para que o valor solicitado fosse o MÁXIMO (e não o mínimo)?


Agradeço de antemão por sua (ou de qualquer outra pessoa

que tenha feito a prova do IME esse ano que passou).


Abraço,

sergio


João Maldonado 
http://www.mail-archive.com/search?l=obm-l@mat.puc-rio.brq=from:%22Jo%C3%A3o+Maldonado%22
Wed, 11 Dec 2013 08:44:26 -0800
http://www.mail-archive.com/search?l=obm-l@mat.puc-rio.brq=date:20131211

Boa Tarde pra todo mundo :)

Eu prestei o IME no mês de outubro e recentemente chegou a prova corrigida no
meu email,
Eu fiquei com nota 9 em matemática, mas jurava que tinha acertado a última
questão (pelo menos a letra A), e a nota que eles deram foi 0.1. Eu tenho até
depois de amanhã pra poder pedir recorreção da questão, queria pedir a ajuda de
vocês, não sei se o que eu escrevi é válido eu não

Obs. a dupla seta significa se e somente se



-- 
Esta mensagem foi verificada pelo sistema de antivírus e
 acredita-se estar livre de perigo.

-- 
Esta mensagem foi verificada pelo sistema de antivírus e
 acredita-se estar livre de perigo.



[obm-l] OBM

2013-11-19 Por tôpico rodrigo pires de araújo
Olá, já faz algum tempo que eu recebo os e-mails sobre a OBM, mas eu não queria 
receber mais. Se puder parar de enviar, eu agradeceria. 
  
-- 
Esta mensagem foi verificada pelo sistema de antivírus e
 acredita-se estar livre de perigo.



Re: [obm-l] OBM-U

2013-02-12 Por tôpico douglas . oliveira
  

Provas anteriores do site da obm. 

On Tue, 12 Feb 2013 06:00:59
+0300, Lucas Azevedo wrote: 

 Quais os livros que são mais indicados
para estudar para a OBM-U? (Sem levar em consideração a bibliografia do
site da OBM) Quais são os assuntos nos quais nós devemos nos focar na
preparação da OBM-U?

  

[obm-l] OBM-U

2013-02-11 Por tôpico Lucas Azevedo

Quais os livros que são mais indicados para estudar para a OBM-U? (Sem levar em 
consideração a bibliografia do site da OBM) Quais são os assuntos nos quais nós 
devemos nos focar na preparação da OBM-U?   
  

[obm-l] Re: [obm-l] OBM Nível III

2012-11-27 Por tôpico terence thirteen
http://www.obm.org.br/export/sites/default/provas_gabaritos/docs/2012/3fase_nivel3_2012.pdf
Bem, quem quiser, vai resolvendo. Eu mesmo vou postar assim:

OBM 2012 - Nível 3 - Problema 1

Vou matutar nesses problemas, e o que eu achar interessante vou postando

Em 26/11/12, Athos Coutoathos...@hotmail.com escreveu:

 Bem, fui fazer a 3ª fase da OBM esse ano.
 Gostei bastante da prova (não quer dizer que eu fui bem, hahaha)
 Gostaria de sugerir a quem tiver disposição:
 Tentem resolver as questões e coloquem as resoluções aqui. Ou, pelo menos,
 dê ideias.
 A prova está disponível no site da OBM (não vou por o link aqui, porque não
 sei se pode por links nessa lista).
   


-- 
/**/
神が祝福

Torres

=
Instru��es para entrar na lista, sair da lista e usar a lista em
http://www.mat.puc-rio.br/~obmlistas/obm-l.html
=


[obm-l] OBM Nível III

2012-11-26 Por tôpico Athos Couto

Bem, fui fazer a 3ª fase da OBM esse ano.
Gostei bastante da prova (não quer dizer que eu fui bem, hahaha)
Gostaria de sugerir a quem tiver disposição:
Tentem resolver as questões e coloquem as resoluções aqui. Ou, pelo menos, dê 
ideias.
A prova está disponível no site da OBM (não vou por o link aqui, porque não sei 
se pode por links nessa lista).
  

[obm-l] obm

2012-10-31 Por tôpico rodrigo pires de araújo

como eu saio da lista?
  

Re: [obm-l] OBM 2011

2012-10-15 Por tôpico Heitor Bueno Ponchio Xavier
Uma ideia legal é tomar o numero chapa C = 55...534343434...34, com k²-r
cincos, r três e r quatros.
Tomando k²= n(k+1)², e 0=r=2k. Tome n= 2k-r+2r e a soma dos digitos de
C é
S(C)=5²(k²-r) + (3²+4²)r=5²k²
acho que é isso

Em 14 de outubro de 2012 10:52, terence thirteen
peterdirich...@gmail.comescreveu:

 Em 14 de outubro de 2012 08:38, Athos Couto athos...@hotmail.com
 escreveu:
  Bem, na verdade são (n+8)!/n!8! somas, até porque 9^n/9! nem número
 inteiro
  (também cheguei a pensar que era isso...) é.
  Realmente, não entendi seus argumentos Bernardo.
  E que teorema é esse? um teorema de Bezout nos afirma que todo natural
  grande pode ser escrito como somas de vários m^2 e n^2.
  Também tentei por indução:

 Se d é o maior divisor comum de a e b, existem inteiros X,Y tal que
 Xa+Yb=d.

 Em especial, se a,b são primos entre si qualquer natural pode ser
 escrito como uma combinação linear de a,b. Para naturais
 suficientemente grandes, eles podem ser escritos usando apenas
 adições.

  Numa sequencia de n-1 números você terá: (n+7)!/(n-1)!8! somas
 diferentes.
  Supomos que alguma(s) seja(m) quadrado(s) perfeito(s).
  Deixe-me quebrar a linha de pensamento:
  Também pensei no seguinte: (m+1)^2 - m^2 = 2m+1
  Se pudéssemos relacionar esse fato com a indução...
  Voltando à indução:
  Formaríamos (n+8)!/n!8! números.
  Na verdade teríamos formado (n+7)!/n!7! a mais do que da última coluna.
  Se alguém conseguir continuar... não consegui ver mais nada. Acho que
 esse
  não é o caminho..
 
  Date: Sun, 14 Oct 2012 01:24:09 -0400
  Subject: Re: [obm-l] OBM 2011
  From: bernardo...@gmail.com
  To: obm-l@mat.puc-rio.br
 
 
  2012/10/13 terence thirteen peterdirich...@gmail.com:
   Eu pensei em alguma indução, mas fala sério, tem que somar com alguma
   propriedade legal.
  
   Se pudéssemos fazer algo com ALPHA*m^2+BETA*n^2, em que m e n são
   primos entre si, um teorema de Bezout nos afirma que todo natural
   grande pode ser escrito como somas de vários m^2 e n^2.
  Que tal contar?
 
  Entre ... e ... as somas dos quadrados dos dígitos variam de n
  a 81n. Por outro lado, as somas possíveis são 9^n / 9! (descontando a
  ordem). Logo deve haver um monte que coincidem. Mas acho que, com um
  pouquinho de sorte, para n suficientement grade, temos praticamente
  todos os valores possíveis entre n e 81n. Deve ter um número aí no
  meio que seja um quadrado. Por exemplo, ([sqrt(n)] + 1)^2 é com
  certeza menor do que 4n para n  1.
 
  Chutando com um computador: para n suficientemente grande, todos os
  números entre n+14 e 64n são factíveis. Provavelmente deve ser até
  melhor do que isso no upper bound. O lower bound é mais fácil: você
  tem um monte de 1. Trocar 1 por 2 aumenta três, 1 por 3 aumenta 8.
  Fazer n+14 = n+8+3+3. n+15 é trocar 1 por 4. Fazer n+13 não dá, porque
  as combinações com 8 e 3 não permitem. Mas o real problema é achar um
  quadrado perto de n, o mais próximo pode ser ainda bm longe.
  Imagine n = 1^2 + 1. O próximo está a 2*sqrt(n) de distância...
  hum, e você pode somar 8 ou 3, e se sqrt(n) é grande o suficiente,
  Bézout, acabou. (Você tem n casas para alterar, e 8 e 3 são maoires do
  que 2, e n  sqrt(n)). O único caso ruim em que o próximo quadrado é
  justamente n+13 (que não podemos fazer), o quadrado seguinte está a
  2*sqrt(n+13) + 1 de distância, e o Bézout garante que podemos fazer
  todas os inteiros entre 7*2 e 8*n - 7*2 (ou algo próximo a isso), se
  tivermos no máximo n termos para escolher entre 3 e 8, e o crescimento
  linear é mais do que suficiente. Agora, basta provar para os casos em
  que n é pequeno, mas a gente já fez!
 
  Quem se aventura a provar que dá pra fazer quase todos os números? Eu
  aposto que tem a ver com a^2 + b^2 = (a-1)^2 + (b+1)^2 + 2(a-b-1).
 
  Abraços,
  --
  Bernardo Freitas Paulo da Costa
 
 
 =
  Instruções para entrar na lista, sair da lista e usar a lista em
  http://www.mat.puc-rio.br/~obmlistas/obm-l.html
 
 =



 --
 /**/
 神が祝福

 Torres

 =
 Instru�ões para entrar na lista, sair da lista e usar a lista em
 http://www.mat.puc-rio.br/~obmlistas/obm-l.html
 =



RE: [obm-l] OBM 2011

2012-10-14 Por tôpico Athos Couto

Bem, na verdade são (n+8)!/n!8! somas, até porque 9^n/9! nem número inteiro 
(também cheguei a pensar que era isso...) é.Realmente, não entendi seus 
argumentos Bernardo.E que teorema é esse? um teorema de Bezout nos afirma que 
todo natural grande pode ser escrito como somas de vários m^2 e n^2.Também 
tentei por indução:Numa sequencia de n-1 números você terá: (n+7)!/(n-1)!8! 
somas diferentes.Supomos que alguma(s) seja(m) quadrado(s) perfeito(s).Deixe-me 
quebrar a linha de pensamento:Também pensei no seguinte: (m+1)^2 - m^2 = 2m+1Se 
pudéssemos relacionar esse fato com a indução...Voltando à indução:Formaríamos 
(n+8)!/n!8! números.Na verdade teríamos formado (n+7)!/n!7! a mais do que da 
última coluna.Se alguém conseguir continuar... não consegui ver mais nada. Acho 
que esse não é o caminho..

 Date: Sun, 14 Oct 2012 01:24:09 -0400
 Subject: Re: [obm-l] OBM 2011
 From: bernardo...@gmail.com
 To: obm-l@mat.puc-rio.br
 
 2012/10/13 terence thirteen peterdirich...@gmail.com:
  Eu pensei em alguma indução, mas fala sério, tem que somar com alguma
  propriedade legal.
 
  Se pudéssemos fazer algo com ALPHA*m^2+BETA*n^2, em que m e n são
  primos entre si, um teorema de Bezout nos afirma que todo natural
  grande pode ser escrito como somas de vários  m^2 e n^2.
 Que tal contar?
 
 Entre ... e ... as somas dos quadrados dos dígitos variam de n
 a 81n. Por outro lado, as somas possíveis são 9^n / 9! (descontando a
 ordem). Logo deve haver um monte que coincidem. Mas acho que, com um
 pouquinho de sorte, para n suficientement grade, temos praticamente
 todos os valores possíveis entre n e 81n. Deve ter um número aí no
 meio que seja um quadrado. Por exemplo, ([sqrt(n)] + 1)^2 é com
 certeza menor do que 4n para n  1.
 
 Chutando com um computador: para n suficientemente grande, todos os
 números entre n+14 e 64n são factíveis. Provavelmente deve ser até
 melhor do que isso no upper bound. O lower bound é mais fácil: você
 tem um monte de 1. Trocar 1 por 2 aumenta três, 1 por 3 aumenta 8.
 Fazer n+14 = n+8+3+3. n+15 é trocar 1 por 4. Fazer n+13 não dá, porque
 as combinações com 8 e 3 não permitem. Mas o real problema é achar um
 quadrado perto de n, o mais próximo pode ser ainda bm longe.
 Imagine n = 1^2 + 1. O próximo está a 2*sqrt(n) de distância...
 hum, e você pode somar 8 ou 3, e se sqrt(n) é grande o suficiente,
 Bézout, acabou. (Você tem n casas para alterar, e 8 e 3 são maoires do
 que 2, e n  sqrt(n)). O único caso ruim em que o próximo quadrado é
 justamente n+13 (que não podemos fazer), o quadrado seguinte está a
 2*sqrt(n+13) + 1 de distância, e o Bézout garante que podemos fazer
 todas os inteiros entre 7*2 e 8*n - 7*2 (ou algo próximo a isso), se
 tivermos no máximo n termos para escolher entre 3 e 8, e o crescimento
 linear é mais do que suficiente. Agora, basta provar para os casos em
 que n é pequeno, mas a gente já fez!
 
 Quem se aventura a provar que dá pra fazer quase todos os números? Eu
 aposto que tem a ver com a^2 + b^2 = (a-1)^2 + (b+1)^2 + 2(a-b-1).
 
 Abraços,
 -- 
 Bernardo Freitas Paulo da Costa
 
 =
 Instruções para entrar na lista, sair da lista e usar a lista em
 http://www.mat.puc-rio.br/~obmlistas/obm-l.html
 =
  

Re: [obm-l] OBM 2011

2012-10-14 Por tôpico terence thirteen
Em 14 de outubro de 2012 08:38, Athos Couto athos...@hotmail.com escreveu:
 Bem, na verdade são (n+8)!/n!8! somas, até porque 9^n/9! nem número inteiro
 (também cheguei a pensar que era isso...) é.
 Realmente, não entendi seus argumentos Bernardo.
 E que teorema é esse? um teorema de Bezout nos afirma que todo natural
 grande pode ser escrito como somas de vários m^2 e n^2.
 Também tentei por indução:

Se d é o maior divisor comum de a e b, existem inteiros X,Y tal que Xa+Yb=d.

Em especial, se a,b são primos entre si qualquer natural pode ser
escrito como uma combinação linear de a,b. Para naturais
suficientemente grandes, eles podem ser escritos usando apenas
adições.

 Numa sequencia de n-1 números você terá: (n+7)!/(n-1)!8! somas diferentes.
 Supomos que alguma(s) seja(m) quadrado(s) perfeito(s).
 Deixe-me quebrar a linha de pensamento:
 Também pensei no seguinte: (m+1)^2 - m^2 = 2m+1
 Se pudéssemos relacionar esse fato com a indução...
 Voltando à indução:
 Formaríamos (n+8)!/n!8! números.
 Na verdade teríamos formado (n+7)!/n!7! a mais do que da última coluna.
 Se alguém conseguir continuar... não consegui ver mais nada. Acho que esse
 não é o caminho..

 Date: Sun, 14 Oct 2012 01:24:09 -0400
 Subject: Re: [obm-l] OBM 2011
 From: bernardo...@gmail.com
 To: obm-l@mat.puc-rio.br


 2012/10/13 terence thirteen peterdirich...@gmail.com:
  Eu pensei em alguma indução, mas fala sério, tem que somar com alguma
  propriedade legal.
 
  Se pudéssemos fazer algo com ALPHA*m^2+BETA*n^2, em que m e n são
  primos entre si, um teorema de Bezout nos afirma que todo natural
  grande pode ser escrito como somas de vários m^2 e n^2.
 Que tal contar?

 Entre ... e ... as somas dos quadrados dos dígitos variam de n
 a 81n. Por outro lado, as somas possíveis são 9^n / 9! (descontando a
 ordem). Logo deve haver um monte que coincidem. Mas acho que, com um
 pouquinho de sorte, para n suficientement grade, temos praticamente
 todos os valores possíveis entre n e 81n. Deve ter um número aí no
 meio que seja um quadrado. Por exemplo, ([sqrt(n)] + 1)^2 é com
 certeza menor do que 4n para n  1.

 Chutando com um computador: para n suficientemente grande, todos os
 números entre n+14 e 64n são factíveis. Provavelmente deve ser até
 melhor do que isso no upper bound. O lower bound é mais fácil: você
 tem um monte de 1. Trocar 1 por 2 aumenta três, 1 por 3 aumenta 8.
 Fazer n+14 = n+8+3+3. n+15 é trocar 1 por 4. Fazer n+13 não dá, porque
 as combinações com 8 e 3 não permitem. Mas o real problema é achar um
 quadrado perto de n, o mais próximo pode ser ainda bm longe.
 Imagine n = 1^2 + 1. O próximo está a 2*sqrt(n) de distância...
 hum, e você pode somar 8 ou 3, e se sqrt(n) é grande o suficiente,
 Bézout, acabou. (Você tem n casas para alterar, e 8 e 3 são maoires do
 que 2, e n  sqrt(n)). O único caso ruim em que o próximo quadrado é
 justamente n+13 (que não podemos fazer), o quadrado seguinte está a
 2*sqrt(n+13) + 1 de distância, e o Bézout garante que podemos fazer
 todas os inteiros entre 7*2 e 8*n - 7*2 (ou algo próximo a isso), se
 tivermos no máximo n termos para escolher entre 3 e 8, e o crescimento
 linear é mais do que suficiente. Agora, basta provar para os casos em
 que n é pequeno, mas a gente já fez!

 Quem se aventura a provar que dá pra fazer quase todos os números? Eu
 aposto que tem a ver com a^2 + b^2 = (a-1)^2 + (b+1)^2 + 2(a-b-1).

 Abraços,
 --
 Bernardo Freitas Paulo da Costa

 =
 Instruções para entrar na lista, sair da lista e usar a lista em
 http://www.mat.puc-rio.br/~obmlistas/obm-l.html
 =



-- 
/**/
神が祝福

Torres

=
Instru��es para entrar na lista, sair da lista e usar a lista em
http://www.mat.puc-rio.br/~obmlistas/obm-l.html
=


[obm-l] OBM 2011

2012-10-13 Por tôpico Athos Couto

Dizemos que um número inteiro positivo é chapa quando ele é formado apenas por 
algarismos não nulos e a soma dos quadrados de todos os seus algarismos é 
também um quadrado perfeito. Prove que, para todo inteiro positivo n, existe um 
número chapa com exatamente n algarismos.
Alguma ideia? 

Re: [obm-l] OBM 2011

2012-10-13 Por tôpico Victor Hugo
3, 4 e 5

Sent from my iPad

On 13/10/2012, at 11:16, Athos Couto athos...@hotmail.com wrote:

 Dizemos que um número inteiro positivo é chapa quando ele é formado apenas 
 por algarismos não nulos e a 
 soma dos quadrados de todos os seus algarismos é também um quadrado perfeito. 
 Prove que, para todo inteiro positivo n, existe um número chapa com 
 exatamente n algarismos.
 
 Alguma ideia?


Re: [obm-l] OBM 2011

2012-10-13 Por tôpico terence thirteen
Em 13 de outubro de 2012 12:53, Victor Hugo
victorhcr.victorh...@gmail.com escreveu:
 3, 4 e 5

 Sent from my iPad

 On 13/10/2012, at 11:16, Athos Couto athos...@hotmail.com wrote:

 Dizemos que um número inteiro positivo é chapa quando ele é formado apenas
 por algarismos não nulos e a
 soma dos quadrados de todos os seus algarismos é também um quadrado
 perfeito.
 Prove que, para todo inteiro positivo n, existe um número chapa com
 exatamente n algarismos.

 Alguma ideia?

Pensando aqui, 3^2+4^2=25, logo 34 é chapa. Não deve ser difícil ver
que 34344433 é chapa, pois a soma é 4*(3^2+4^2).

Assim, para todo N múltiplo de 8, existe um chapa de N algarismos.
Difícil vai ser cobrir os restantes... :(

-- 
/**/
神が祝福

Torres

=
Instru��es para entrar na lista, sair da lista e usar a lista em
http://www.mat.puc-rio.br/~obmlistas/obm-l.html
=


Re: [obm-l] OBM 2011

2012-10-13 Por tôpico Bernardo Freitas Paulo da Costa
 Pensando aqui, 3^2+4^2=25, logo 34 é chapa. Não deve ser difícil ver
 que 34344433 é chapa, pois a soma é 4*(3^2+4^2).

 Assim, para todo N múltiplo de 8, existe um chapa de N algarismos.
 Difícil vai ser cobrir os restantes... :(
Eu sei fazer com 4, 5, 6, e 7 algarismos, roubando a idéia do Torres.
Só não sei como ele faz com 16 algarismos, já que 8*(3^2 + 4^2) não é
um quadrado... Você provou que dá pra fazer todos os 8N^2, na verdade,
com 3 e 4, você faz todos os 2n^2 com o mesmo argumento. Usando que
13^2 = 5^2 + 12^2 (12 = 2*6, ou 3*4, ou 4*3, ou 6*2) você deve
conseguir um monte de outros alfa*n^2. Mas o problema é que quadrados
são mutio raros nos números inteiros...
-- 
Bernardo Freitas Paulo da Costa

=
Instruções para entrar na lista, sair da lista e usar a lista em
http://www.mat.puc-rio.br/~obmlistas/obm-l.html
=


Re: [obm-l] OBM 2011

2012-10-13 Por tôpico terence thirteen
l;

Em 13 de outubro de 2012 21:28, Bernardo Freitas Paulo da Costa
bernardo...@gmail.com escreveu:
 Pensando aqui, 3^2+4^2=25, logo 34 é chapa. Não deve ser difícil ver
 que 34344433 é chapa, pois a soma é 4*(3^2+4^2).

 Assim, para todo N múltiplo de 8, existe um chapa de N algarismos.
 Difícil vai ser cobrir os restantes... :(
 Eu sei fazer com 4, 5, 6, e 7 algarismos, roubando a idéia do Torres.
 Só não sei como ele faz com 16 algarismos, já que 8*(3^2 + 4^2) não é
 um quadrado... Você provou que dá pra fazer todos os 8N^2, na verdade,
 com 3 e 4, você faz todos os 2n^2 com o mesmo argumento. Usando que
 13^2 = 5^2 + 12^2 (12 = 2*6, ou 3*4, ou 4*3, ou 6*2) você deve
 conseguir um monte de outros alfa*n^2. Mas o problema é que quadrados
 são mutio raros nos números inteiros...

Eu pensei em alguma indução, mas fala sério, tem que somar com alguma
propriedade legal.

Algo como se X^2+Y^2 é quadrado, e X^2 é soma de quadrados de alguns
caras, enquanto Y^2 também é, podemos juntar os caras. Por exemplo,
5^2+12^2=13^2, assim encontrando caras cujos quadrados somam 144 = por
exemplo 9731, basta juntar um 5 nisso e obtemos um cara de 9
dígitos. Outro seria 884, a soma dos quadrados dá 12^2. Igualmente,
8845 daria 4 dígitos

Se pudéssemos fazer algo com ALPHA*m^2+BETA*n^2, em que m e n são
primos entre si, um teorema de Bezout nos afirma que todo natural
grande pode ser escrito como somas de vários  m^2 e n^2.


 --
 Bernardo Freitas Paulo da Costa

 =
 Instruções para entrar na lista, sair da lista e usar a lista em
 http://www.mat.puc-rio.br/~obmlistas/obm-l.html
 =



-- 
/**/
神が祝福

Torres

=
Instru��es para entrar na lista, sair da lista e usar a lista em
http://www.mat.puc-rio.br/~obmlistas/obm-l.html
=


Re: [obm-l] OBM 2011

2012-10-13 Por tôpico Bernardo Freitas Paulo da Costa
2012/10/13 terence thirteen peterdirich...@gmail.com:
 Eu pensei em alguma indução, mas fala sério, tem que somar com alguma
 propriedade legal.

 Se pudéssemos fazer algo com ALPHA*m^2+BETA*n^2, em que m e n são
 primos entre si, um teorema de Bezout nos afirma que todo natural
 grande pode ser escrito como somas de vários  m^2 e n^2.
Que tal contar?

Entre ... e ... as somas dos quadrados dos dígitos variam de n
a 81n. Por outro lado, as somas possíveis são 9^n / 9! (descontando a
ordem). Logo deve haver um monte que coincidem. Mas acho que, com um
pouquinho de sorte, para n suficientement grade, temos praticamente
todos os valores possíveis entre n e 81n. Deve ter um número aí no
meio que seja um quadrado. Por exemplo, ([sqrt(n)] + 1)^2 é com
certeza menor do que 4n para n  1.

Chutando com um computador: para n suficientemente grande, todos os
números entre n+14 e 64n são factíveis. Provavelmente deve ser até
melhor do que isso no upper bound. O lower bound é mais fácil: você
tem um monte de 1. Trocar 1 por 2 aumenta três, 1 por 3 aumenta 8.
Fazer n+14 = n+8+3+3. n+15 é trocar 1 por 4. Fazer n+13 não dá, porque
as combinações com 8 e 3 não permitem. Mas o real problema é achar um
quadrado perto de n, o mais próximo pode ser ainda bm longe.
Imagine n = 1^2 + 1. O próximo está a 2*sqrt(n) de distância...
hum, e você pode somar 8 ou 3, e se sqrt(n) é grande o suficiente,
Bézout, acabou. (Você tem n casas para alterar, e 8 e 3 são maoires do
que 2, e n  sqrt(n)). O único caso ruim em que o próximo quadrado é
justamente n+13 (que não podemos fazer), o quadrado seguinte está a
2*sqrt(n+13) + 1 de distância, e o Bézout garante que podemos fazer
todas os inteiros entre 7*2 e 8*n - 7*2 (ou algo próximo a isso), se
tivermos no máximo n termos para escolher entre 3 e 8, e o crescimento
linear é mais do que suficiente. Agora, basta provar para os casos em
que n é pequeno, mas a gente já fez!

Quem se aventura a provar que dá pra fazer quase todos os números? Eu
aposto que tem a ver com a^2 + b^2 = (a-1)^2 + (b+1)^2 + 2(a-b-1).

Abraços,
-- 
Bernardo Freitas Paulo da Costa

=
Instruções para entrar na lista, sair da lista e usar a lista em
http://www.mat.puc-rio.br/~obmlistas/obm-l.html
=


  1   2   3   4   5   6   >